Вы находитесь на странице: 1из 82

Apuntes de

Teora Elemental de N
umeros
por Enrique Arrondo(*)

Version del 19 de Enero de 2009


Aunque en orden diverso, estas notas est
an basadas en el libro Elementary Number Theory, de David
M. Burton, donde el lector puede profundizar en los detalles.

0. Preliminares
1. Divisibilidad y factorizaci
on en enteros
2. Teora de congruencias
3. Funciones aritmeticas

4. Ordenes,
races primitivas e ndices
5. Congruencias cuadr
aticas
6. Ecuaciones diof
anticas
7. Fracciones continuas

(*) Departamento de Algebra,


Facultad de Ciencias Matem
aticas, Universidad Complutense de Madrid, 28040 Madrid, arrondo@mat.ucm.es
1

0. Preliminares
Denici
on. Denotaremos por Z al conjunto de n
umeros enteros. Dado a Z, denotaremos
umeros enteros mayores o iguales que a. De esta forma
por Za al conjunto de los n
evitaremos polemicas esteriles sobre si los n
umeros naturales empiezan con el 0 o con el 1,
un nos haga falta.
y escribiremos simplemente Z0 o Z1 seg
Teorema 0.1 (Principio del buen orden). Sea a un n
umero entero y sea S
subconjunto no vaco de Za . Entonces S tiene un elemento mnimo.

Za

un

Demostraci
on: Como S es no vaco, existe b S. Como S Za , entonces a b.
Considerando el conjunto nito {a, a + a, . . . , b 1, b}, tomamos c el menor elemento de tal
conjunto que este en S (sabemos que, en el peor de los casos, b lo esta). Es claro entonces
que c es un elemento mnimo de S.
Teorema 0.2 (Principio de inducci
on). Sea P una propiedad relativa a los n
umeros de
Za . Supongamos que se verica alguna de las dos siguientes hipotesis:
(i) (Inducci
on debil): P es cierta para el n
umero a y cada vez que P es cierta para cada
n Za entonces P es cierta para n + 1.
(ii) (Inducci
on fuerte): P es cierta para el n
umero a y cada vez que P es cierta para todos
los n
umeros de a a n entonces P es cierta para n + 1.
Entonces P es cierta para todo n Za .
Demostraci
on: Sea S el conjunto de enteros mayores o iguales que a que no verica la
propiedad P . Como estamos suponiendo que a verica P , entonces a S. El teorema
estara demostrado si demostramos que S es el conjunto vaco. Supongamos entonces, por
reducci
on al absurdo, que S es no vaco. Por el principio del buen orden, el conjunto S
tendr
a un mnimo m. Esto quiere decir que los n
umeros a, a + 1, . . . , m 1 no estan en S,
es decir, verican la propiedad P . Pero tanto (i) como (ii) aplicadas a n = m 1 implican
que entonces m tambien verica P , lo que contradice que m este en S.
Ejercicio 0.3. Se considera la sucesi
on de Fibonacci {un } de n
umeros enteros
1, 1, 2, 3, 5, 8, 13, 21, . . .
on
denida de formarecursiva por
u1 = u2 = 1, un = un1 + un2 . Probar por inducci

n
n
1
1+
5
1
5
que un = 5
. Comprobar la f
ormula para n = 4.

2
2

1. Divisibilidad y factorizaci
on en enteros
Denici
on. Se dice que un n
umero entero a divide a otro n
umero entero b y se escribe
a|b si existe c Z tal que b = ac. En tal caso, diremos que a es un divisor de b.
Demostramos a continuacion las propiedades b
asicas de divisibilidad que necesitaremos. Las demostraciones son extremadamente simples, pero vale la pena hacerlas con
rigor al menos una vez, ya que veremos enseguida que hay propiedades aparentemente
inmediatas (en el sentido de que son propiedades de sobra conocidas) y que, sin embargo,
no pueden hacerse a partir de la denici
on y requieren tecnicas mas sosticadas.
Lema 1.1. Dados enteros a, b, c, se tiene:
(i) Si a|b y b = 0, entonces |a| |b|.
(ii) c|a y c|b, entonces c|ax + by para todo x, y Z.
(iii) a|b si y solo si a|ac + b.
(iv) Si c = 0, entonces a|b si y solo si ac|bc.
Demostraci
on: Demostremos primero (i). Si a|b, entonces existe un entero c tal que b = ac.
Tomando valores absolutos, se tiene |b| = |a||c|. Como b = 0, se tiene c = 0, luego |c| 1.
Por tanto, |b| |a|.
Para demostrar (ii), vemos que c|a y c|b implica que existen enteros a , b tales que
a = a c y b = b c. Por tanto, ax + by = a cx + b cy = (a x + b y)c, por lo que c|ax + by.
La parte (iii) es consecuencia de (ii). En efecto, si a|b, como obviamente a|a, se tiene
a|ac + b 1 = ac + b. Recprocamente, si a|ac + b, de nuevo por (ii) se tiene a|(ac + b) 1 +
a(c) = b.
Finalmente, para ver (iv), por denici
on tenemos que a|b si y solo si existe un entero
d tal que b = ad. Como c = 0, esto es equivalente a bc = acd, que de nuevo equivale, por
denici
on, a ac|bc.
Denici
on. Un n
umero primo es un entero p > 1 cuyos u
nicos divisores positivos son 1
y p. Por el contrario, un n
umero compuesto es un n
umero entero n > 1 con alg
un divisor
positivo n1 = 1, n, y por tanto n = n1 n2 , con n1 , n2 > 1.
Empezamos con una aplicaci
on del principio de inducci
on fuerte:
Teorema 1.2 (Fundamental de la aritmetica: existencia). Todo n
umero entero n 2 se
puede escribir como producto nito de n
umeros primos.
Demostraci
on: Lo demostraremos por inducci
on sobre n Z2 . Claramente 2 es primo,
luego el teorema se verica trivialmente para n = 2. Supongamos ahora que tenemos
3

demostrado el teorema para todos los n


umeros 2, 3, . . . , n y veamos que tambien es cierto
para n + 1. Si n + 1 fuera un n
umero primo, de nuevo el resultado sera trivial. Queda
entonces demostrarlo en el caso en que n + 1 no es primo. En tal caso, n + 1 tendra un
divisor positivo a distinto de 1 y n + 1. Es decir, existe b Z tal que n + 1 = ab. Como
n + 1 y a son positivos, tambien lo es b. Ademas, como a = 1, n + 1, tambien se tiene
b = 1, n + 1. De aqu se deduce, junto al Lema 1.1(i), que a y b estan en el conjunto
{2, 3, . . . , n} en que sabemos que es cierto el teorema. Por tanto, a y b se escriben como
producto nito de n
umeros primos, de lo que se sigue que tambien n + 1 = ab es producto
nito de n
umeros primos, como queramos demostrar.
Observaci
on 1.3. Observese que de momento no se puede demostrar que la descomposicion en primos sea u
nica. En efecto, si tenemos que n = p1 . . . pr = p1 . . . ps , no
podemos demostrar con la mera denici
on de n
umero primo que las dos factorizaciones
son iguales (salvo el orden de los factores). La estrategia sera decir que, por ejemplo,
nado que si
p1 divide a n, es decir al producto p1 . . . pr . Pero, aunque nos hayan ense
un n
umero primo p1 divide a un producto de n
umeros p1 . . . pr entonces divide a alguno
de los factores, no podemos usarlo porque de momento s
olo podemos usar la denici
on
de n
umero primo, que no implica nada de esto de forma inmediata. S
olo mas adelante,
cuando demostremos el Lema de Euclides (Teorema 1.13) podremos demostrar la unicidad. De momento, usemos la existencia para demostrar otro resultado importante debido
a Euclides:
Teorema 1.4 (Euclides). Existen innitos n
umeros primos.
Demostraci
on: Supongamos, por reducci
on al absurdo, que el conjunto de n
umeros primos
umero n = p1 p2 . . . pr + 1. Claramente n 2
es un conjunto nito {p1 , p2 , . . . , pr }. Sea el n
(ya que alg
un pi es 2), por lo que el Teorema 1.2 implica que n se escribe como producto
de n
umeros primos. En particular, n tiene alg
un factor primo, que necesariamente ser
a de
un i = 1, 2, . . . , r. Es decir, pi |n = p1 p2 . . . pr + 1. Por el Lema 1.1(iii),
la forma pi , para alg
pi |1, lo que es absurdo.
Denici
on. Se llama m
aximo com
un divisor de dos n
umeros a, b Z (y lo denotaremos
por mcd(a, b)) al mayor divisor positivo de a y b. Dado que esta denici
on no tiene sentido
si a = b = 0 (ya que cualquier n
umero positivo divide a 0), se puede convenir en escribir
mcd(0, 0) = 0, puesto que mcd(a, 0) = a para cualquier a = 0. Si (a, b) = (0, 0), entonces
es claro que mcd(a, b) 1 (por tanto, siempre que demos por descontado que el m
aximo
com
un divisor de dos n
umeros es distinto de cero es porque excluimos el caso trivial en que
ambos n
umeros son cero). Diremos que dos n
umeros a y b son primos entre s o coprimos
si mcd(a, b) = 1.
4

Como ya hemos observado, con la mera denici


on de primos y divisibilidad no se
puede andar muy lejos. Para el caso del m
aximo com
un divisor, sin saber que los n
umeros
se pueden descomponer en primos, no habra siquiera en principio un modo de calcularlo.
Enunciamos en primer lugar los resultados que s pueden hacerse sin mas que la propia
denici
on.
Proposici
on 1.5. El m
aximo com
un divisor verica las siguientes propiedades:
(i) mcd(a, an + b) = mcd(a, b) para cualquier n Z.
(ii) Si d = mcd(a, b), entonces mcd( ad , db ) = 1.
(iii) Dado un n
umero primo p y un entero cualquiera a, entonces o bien p|a o bien
mcd(a, p) = 1.
Demostraci
on: Para demostrar la parte (i) bastar
a que los divisores comunes de a y an + b
son lo divisores comunes de a y b. Tomemos entonces un divisor d|a y veamos que d|an + b
si y s
olo si d|b. En efecto, como d|a, podremos escribir a = da . El Lema 1.1(iii) nos dice
ahora que d divide a an + b = d(a n) + b si y solo si d divide a b.
Para la parte (ii), si fuera c = mcd( ad , db ) > 1, entonces, como c| ad y c| db , se seguira
cd|a y cd|b. Como cd > d, se llega a contradicci
on.
La parte (iii) es inmediata de la denici
on de n
umero primo, ya que, por ser mcd(a, p)
un divisor de p, se tiene que o bien mcd(a, p) = 1 o bien mcd(a, p) = p (en cuyo caso p|a).

El primer resultado importante es el siguiente:


Teorema 1.6 (Algoritmo de divisi
on). Dados a, b
tales que a = qb + r y 0 r < b.

Z con b > 0, existen q, r Z unicos

Demostraci
on: Consideremos el conjunto S = {a xb | x Z, a xb 0}. Como para
x = |a| se tiene a xb = a + |a|b a + |a| 0, el conjunto S es no vaco. Por el Principio
del Buen Orden (usando S Z0 ) se deduce que S tiene un elemento mnimo r = a qb.
Por denici
on, r 0. Veamos que tambien r < b. En efecto, si fuera r b, entonces
tendramos 0 r b = a (q + 1)b, luego r b sera un elemento de S menor que r, lo
que es absurdo. Esto demuestra la existencia de la divisi
on.
Para ver la unicidad, supongamos que tenemos dos divisiones distintas qb + r = a =
q b + r con 0 r, r < b. Entonces se tendra (q q )b = r r, es decir, b|r r. Ahora
bien, r r < b 0 = b y r r < b 0 = b, por lo que |r r| < b = |b|. El Lema 1.1(i)
implica entonces r r = 0, de donde se deduce r = r y q = q .
5

Observaci
on 1.7. El resultado anterior nos permite clasicar los n
umeros seg
un el resto
que den al dividir por un n
umero dado. Por ejemplo, dividiendo por 4, es claro que todo
entero es de alguna de las siguientes formas: 4k, 4k + 1, 4k + 2, 4k + 3. Claramente,
los n
umeros de la forma 4k nunca seran primos, y el u
nico n
umero primo de la forma
4k + 2 es p = 2. Por tanto, seg
un el Teorema 1.4, habr
a innitos primos de la forma
4k + 1 o de la forma 4k + 3 (veremos mas adelante que hay innitos de cualquiera de las
dos formas). Veamos de momento que existen innitos n
umeros primos de la forma 4k + 3
(mas en general, un teorema de Dirichlet asegura que, dados enteros a, b con mcd(a, b) = 1,
existen innitos n
umeros primos de la forma ak + b; la demostracion de este teorema
requiere tecnicas mucho mas avanzadas que las presentes en estas notas, por lo que solo
demostraremos casos muy particulares). El lector reconocera aqu (y en demostraciones
de resultados parecidos) la tecnica de la demostracion de Euclides para la existencia de
innitos n
umeros primos (Teorema 1.4). Si suponemos que p1 , p2 , . . . , pr son todos los
n
umeros primos de la forma 4k + 3, consideramos el n
umero n = 4p1 p2 . . . pr 1. Como
al menos alg
un pi es 3, se tiene n 2. Por el Teorema Fundamental de la Aritmetica
(Teorema 1.2), n sera producto nito de n
umeros primos (necesariamente distintos de 2,
porque n es impar). No pueden ser todos de la forma 4k + 1, porque entonces su producto
n sera tambien de la forma 4k + 1, mientras que n es de la forma 4k + 3. Por tanto, n
es divisible por alg
un n
umero primo de la forma 4k + 3, es decir, es divisible por alg
un pi .
Por el Lema 1.1(i), pi | 1, lo que es absurdo.
Teorema 1.8 (Bezout). Dados a, b Z, entonces el conjunto {ax + by Z | x, y Z}
es el conjunto de los m
ultiplos de d = mcd(a, b). En particular, mcd(a, b) = 1 si y solo si
existen x, y Z tales que ax + by = 1.
Demostraci
on: Podemos suponer que a y b son distintos de cero, ya que en caso contrario
el resultado es inmediato. Observese que, como d|a, b, entonces el Lema 1.1(ii) implica
d|ax + by para todo x, y Z. Basta entonces demostrar que cualquier m
ultiplo de d se
escribe de la forma ax + by, para lo cual es suciente verlo para d.
Consideramos el conjunto S = {ax + by | x, y Z, ax + by 1}. Como a2 + b2 > 0,
se tiene que S es no vaco, ya que contiene al menos a a2 + b2 . Por el Principio del Buen
Orden (viendo S como subconjunto de Z1 ), se sigue que S tiene un elemento mnimo, que
sera de la forma d = ax + by. Queremos ver que d = d, lo que terminara la demostracion.
on eucldea y tendremos a =
Veamos primero que d |a. Para ello efectuamos la divisi
qd + r, con 0 r < d . Escribimos entonces
r = a qd = a q(ax + by) = a(1 q) + b(qy).
Como r < d y d es minimo en S, no puede ser que r este en S, por lo que la igualdad
anterior implica r 0, luego r = 0 y por tanto d |a. De forma an
aloga se demuestra d |b.
6

Basta ver, por la denici


on de m
aximo com
un m
ultiplo, que cualquier divisor com
un
positivo de a y b es menor o igual que d . Sea entonces c > 0 tal que c|a y c|b. Se tiene
entonces, por el Lema 1.1(ii) que c|ax + by = d , luego por el Lema 1.1(i) c d , como
queramos.
El resultado anterior nos permite obtener un gran n
umero de propiedades de la divisibilidad que seran obvias usando factorizaci
on en primos pero que no son obvias usando
solo la denici
on de divisibilidad.
Proposici
on 1.9. Dados n
umeros enteros no nulos a1 , a2 , b, se tiene que mcd(a1 a2 , b) = 1
si y solo si mcd(a1 , b) = 1 y mcd(a2 , b) = 1.
Demostraci
on: Es claro, usando s
olo divisibilidad, que si mcd(a1 a2 , b) = 1, entonces a1 y
b no pueden tener un divisor com
un mayor de uno (porque sera un divisor com
un de a1 a2
y b), ni tampoco a2 y B pueden tenerlo, por lo que mcd(a1 , b) = 1 y mcd(a2 , b) = 1.
Recprocamente, si mcd(a1 , b) = 1 y mcd(a2 , b) = 1, por el Teorema 1.8 se sigue que
existen x1 , y1 , x2 , y2 Z tales que
a1 x1 + by1 = 1
a2 x2 + by2 = 1.
Multiplicando ambas expresiones, se obtiene
a1 a2 x1 x2 + b(a1 x1 y2 + a2 x2 y1 + by1 y2 ) = 1,
lo que implica, de nuevo por el Teorema 1.8, mcd(a1 a2 , b) = 1.
Muchas veces que tengamos un resultado como el anterior, v
alido para dos elementos
alido tambien para un n
umero nito de elementos, demostrandose este hecho
a1 , a2 , sera v
por recurrencia (que en esencia es lo mismo que la induccion debil). Para ilustrar este
hecho, demostramos a continuaci
on la generalizaci
on de la Proposici
on 1.9 a r factores
con el lenguaje de la inducci
on, mientras que en el Corolario 1.12 haremos lo mismo a
partir de la Proposici
on 1.11, usando en ese caso el metodo de inducci
on. Queda al lector
el convencerse de que en realidad ambos metodos son equivalentes, y escoger cual de los
dos le resulta mas convincente para aplicarlo en casos sucesivos (ya que a partir de ahora
enunciaremos sin demostrar resultados de este tipo).
Corolario 1.10. mcd(a1 a2 . . . ar , b) = 1 si y solo si mcd(ai , b) = 1 para todo i =
1, 2, . . . , r.
on 1.9 tenDemostraci
on: Si agrupamos a1 a2 . . . ar = (a1 a2 . . . ar1 )ar , por la Proposici
dremos que mcd(a1 a2 . . . ar , b) = 1 si y s
olo si mcd(a1 a2 . . . ar1 , b) = 1 y mcd(ar , b) = 1.
7

Repetimos lo mismo para a1 a2 . . . ar1 y tendremos ahora que mcd(a1 a2 . . . ar1 , b) = 1 si


y solo si mcd(a1 a2 . . . ar2 , b) = 1 y mcd(ar1 , b) = 1. Por tanto, mcd(a1 a2 . . . ar , b) = 1
si y s
olo si mcd(a1 a2 . . . ar2 , b) = 1, mcd(ar1 , b) = 1 y mcd(ar , b) = 1. Est
a claro que,
reiterando el procedimiento, se llega al enunciado.
Proposici
on 1.11. Dados a, b, n
entonces ab|n.

tales que mcd(a, b) = 1 y a|n, b|n, se tiene que

Demostraci
on: De la hip
otesis a|n, b|n podemos escribir n = ar = bs, con r, s Z. Por
otra parte, como mcd(a, b) = 1, el Teorema 1.8 implica que existen x, y Z tales que
ax + by = 1. Tenemos entonces
n = n 1 = n(ax + by) = nax + nby = (bs)ax + (ar)by = ab(sx + ry),
por lo que ab|n.
Corolario 1.12. Sean a1 , a2 , . . . , ar n
umeros enteros tales que mcd(ai , aj ) = 1 si i = j.
Entonces, dado un n
umero entero n tal que ai |n para cada i = 1, 2, . . . , r, se tiene que
a1 a2 . . . ar |n.
Demostraci
on: Lo demostramos por inducci
on sobre el n
umero de factores r. Si r = 2,
entonces es la Proposicion 1.11. Supongamos entonces, por hip
otesis de induccion, que
sabemos que, si r n
umeros coprimos dos a dos dividen a un entero, entonces el producto
tambien divide a ese n
umero. Veamoslo ahora para r + 1 n
umeros coprimos entre s.
Suponemos entonces que tenemos a1 , a2 , . . . , ar+1 tales que mcd(ai , aj ) = 1 si i = j
y supongamos ai |n para cada i = 1, 2, . . . , r + 1. Por hip
otesis de induccion, como en
particular ai |n para cada i = 1, 2, . . . , r, se tiene a1 a2 . . . ar |n. Ademas, por el Corolario
on 1.11 con a =
1.10, se tiene mcd(a1 a2 . . . ar , ar+1 ) = 1, luego aplicando la Proposici
a1 a2 . . . ar y b = ar+1 , se tiene a1 a2 . . . ar ar+1 |n, como queramos.
Teorema 1.13 (Lema de Euclides). Dados a, b, c
tiene que entonces a|c.

Z tales que a|bc y mcd(a, b) = 1, se

Demostraci
on: Como mcd(a, b) = 1, por el Teorema 1.8 existen x, y Z tales que ax+by =
1. Por tanto
c = c 1 = c(ax + by) = a(cx) + (bc)y.
Como a|bc, se sigue que a divide a los dos sumandos de la derecha, luego a|c.
Este resultado nos permite demostrar ya la propiedad clave que todos conocemos de
los n
umeros primos.
8

Corolario 1.14. Sea p un n


umero primo. Si a1 , a2 , . . . , ar son n
umeros enteros tales que
p|a1 a2 . . . ar , entonces existe i = 1, 2, . . . , r tal que p|ai .
Demostraci
on: Supongamos, por reducci
on al absurdo, que, para cada i = 1, 2, . . . , r,
ocurriera p |ai . Por el Lema 1.5(iii), se tendra mcd(p, ai ) = 1, luego, por el Corolario
1.10, mcd(p, a1 a2 , . . . , ar ) = 1. Aplicando el Lema de Euclides con a = p, b = a1 a2 . . . ar y
c = 1, se llegara a p|1, lo que es absurdo.
Con esta propiedad podemos demostrar nalmente por completo el teorema fundamental de la aritmetica (ver Observacion 1.3).
Teorema 1.15. (Fundamental de la aritmetica). Todo n
umero entero n 2 se puede
escribir de forma u
nica (salvo el orden de los factores) como producto nito de n
umeros
primos.
Demostraci
on: La existencia ya la demostramos en el Teorema 1.2, as que solo hay que demostrar la unicidad de la descomposici
on. Supongamos que tenemos dos descomposiciones
n = p1 p2 . . . pr = p1 p2 . . . ps de n en producto de n
umeros primos. Como ps |p1 p2 . . . ps =
un pi . Reordenando los primos
p1 p2 . . . pr , el Corolario 1.14 implica que ps divide a alg
p1 , p2 , . . . , pr , podemos suponer ps |pr . Como pr es primo, se sigue pr = ps . Por tanto,
cancelando este factor en la igualdad anterior, tendremos ps |p1 p2 . . . ps1 = p1 p2 . . . pr1 .
Podemos reiterar el proceso demostrando que cada pj es un pi y cancelandolos en la igualdad. N
otese que no puede ocurrir que se acaben primero los pj (es decir r > s), ya que
entonces llegaramos a una igualdad p1 p2 . . . prs = 1, lo que es imposible. De la misma
forma, no se pueden acabar primero los pi (es decir r < s) ya que en tal caso se llegara al
umero de factores es el mismo, y estos coinciden
absurdo p1 p2 . . . psr = 1. Por tanto, el n
salvo el orden (adem
as cada factor primo aparece el mismo n
umero de veces).
Notaci
on. En general, agruparemos los n
umeros primos que se repitan en la descoma1 a2
ar
posicion y escribiremos n = p1 p2 . . . pr , con p1 , p2 , . . . , pr todos distintos entre s. De
hecho, mientras no especiquemos lo contrario, siempre que digamos que n = pa1 1 pa2 2 . . . par r
es la factorizacion en primos de n, se entender
a que p1 , p2 , . . . , pr todos distintos entre s.
Con los resultados vistos hasta ahora podemos recuperar los metodos ya conocidos de
calcular el maximo com
un divisor.
Proposici
on 1.16. Sea a = pa1 1 pa2 2 . . . par r la descomposicion de un entero a 2 en
producto de n
umeros primos. Entonces los divisores positivos de a son los n
umeros de
c1 c2
cr
la forma p1 p2 . . . pr , con 0 ci ai para cada i = 1, 2, . . . , r. Por tanto, si b =
pb11 pb22 . . . pbrr , se tendr
a mcd(a, b) = pc11 pc22 . . . pcrr , con ci = min{ai , bi } para cada i =
1, 2, . . . , r.
9

Demostraci
on: Es claro que los n
umeros de la forma pc11 pc22 . . . pcrr , con 0 ci ai ,
son divisores positivos de a. Recprocamente, sea d un divisor positivo de a. Existir
a
por tanto otro entero q, tambien positivo, tal que a = qd. Si q = 1 o d = 1 el resultado es trivial, as que supondremos q, d 2. Por el Teorema Fundamental de la
a
a
a
a
a
a
Aritmetica podremos escribir q = p1 1 p2 2 . . . ps s y d = p1 1 p2 2 . . . pt t . Por tanto,
a
a
a
a
a
a
a = p1 1 p2 2 . . . ps s p1 1 p2 2 . . . pt t es una descomposicion de a en factores primos (en
que puede haber primos repetidos). Por la unicidad demostrada en el Teorema 1.15, necea
a
a
sariamente los factores de p1 1 p2 2 . . . pt t son parte de los factores de pa1 1 pa2 2 . . . par r . Esto
a
a
a
quiere decir que p1 1 p2 2 . . . pt t = pc11 pc22 . . . pcrr , con 0 ci ai , lo que demuestra el
resultado.
Corolario 1.17. Sean a, b enteros positivos tales que mcd(a, b) = 1. Entonces:
(i) Los divisores positivos de ab son los productos de la forma dd , con d|a y d |b y d, d > 0.
un c Z1 , entonces a = un y b = v n para ciertos u, v Z.
(ii) Si ab = cn para alg
Demostraci
on: Como mcd(a, b) = 1, en las factorizaciones en factores primos
a = pk11 pk22 . . . pkr r
b = q1l1 q2l2 . . . qsls
se tiene pi = qj para todo i = 1, 2, . . . , r y j, 1, 2, . . . , s. Por tanto, la factorizaci
on en
factores primos de ab es
ab = pk11 pk22 . . . pkr r q1l1 q2l2 . . . qsls ,
luego los divisores positivos de ab son, por la Proposici
on 1.16, los n
umeros de la forma
k1 k2
k r l 1 l2
ls
p1 p2 . . . pr q1 q2 . . . qs , con 0 ki ki y 0 lj lj , es decir, el producto de un divisor
positivo de a y un otro de b. Esto demuestra (i).
Para la parte (ii), sea p un primo que divide a c. En particular, p|cn = ab =
pk11 pk22 . . . pkr r q1l1 q2l2 . . . qsls . Por el Corolario 1.14, se sigue que p es un pi o un qj , luego la
k

factorizacion en primos de c se escribe de la forma c = p11 p22 . . . pr r q11 q22 . . . qss . Por tanto,
nk nk
nk nl nl
nl
pk11 pk22 . . . pkr r q1l1 q2l2 . . . qsls = ab = cn = p1 1 p2 2 . . . pr r q1 1 q2 2 . . . qs s . Por la unicidad del
Teorema 1.15, se sigue que ki = nki para i = 1, 2, . . . , r y lj = nlj para j = 1, 2, . . . , s. Por
k

tanto a = (p11 p22 . . . pr r )n y b = (q11 q22 . . . qss )n .


Teorema 1.18. Sean a, b, c enteros positivos tales que an = bn c para alg
un entero positivo
n
n. Entonces c = d para alg
un d Z. En particular, si c no es la potencia n-esima de un

n
n
umero entero, entonces c no es un n
umero racional.
10

Demostraci
on: Si a = pk11 pk22 . . . pkr r es la descomposicion de a en factores primos, entonces
la descomposicion de an en factores primos sera
1 nk2
r
. . . pnk
an = pnk
r .
1 p2

on 1.16, la descomposici
on de b en factores primos
Como b|an , entonces, por la Proposici
sera de la forma
k k
k
b = p11 p22 . . . pr r
con 0 ki nki para i = 1, 2, . . . , r. De la misma forma, tendremos
k

c = p11 p22 . . . pr r
con 0 ki nki para i = 1, 2, . . . , r. Por tanto,
nk1 +k1

an = bn c = p1

nk2 +k2

p2

nkr +kr

. . . pr

y, por la unicidad de la descomposici


on en primos, para cada i = 1, 2 . . . , r tendremos
nki = nki + ki . Por tanto, ki = n(ki ki ) 0, es decir, ki ki 0. Luego el n
umero
k1 k1 k2 k2
kr kr
n
p2
. . . pr
es un entero positivo que verica d = c.
d = p1

Para la u
ltima parte, si n c fuera racional, se poda escribir n c = ab , con a, b enteros
on
positivos. Por tanto, an = bn c, y por la parte ya demostrada llegaramos a la contradicci
que c debe ser una potencia n-esima de un n
umero entero.
Para el calculo del m
aximo com
un divisor no siempre es pr
actico usar la Proposici
on
1.16 (que requiere, por ejemplo, saber factorizar los n
umeros en cuestion). Usaremos en
cambio muchas veces el metodo mas clasico:
Teorema 1.19 (Algoritmo de Euclides). Sean a, b n
umeros enteros con b > 0. Denimos
por recurrencia r1 = a, r0 = b y, supuestos denidos rk1 , rk con rk = 0, denimos rk+1
como el resto de la division eucldea de de rk1 entre rk . Entonces, existe alg
un n tal que
rn = 0, y se verica ademas que rn1 = mcd(a, b).
Demostraci
on: De la denici
on por recurrencia se sigue que 0 rk+1 < rk para todo
on estrictamente decreciente de enteros no
k 0. Por tanto r0 , r1 , . . . forma una sucesi
negativos, por lo que tiene que ocurrir rn = 0 para alg
un miembro de la sucesion. Por
otra parte, si para cada k = 0, . . . , n escribimos la divisi
on rk1 = qk rk + rk+1 , por la
Proposici
on 1.5(i) se tendr
a mcd(rk1 , rk ) = mcd(qk rk + rk+1 , rk ) = mcd(rk+1 , rk ). Por
tanto:
mcd(a, b) = mcd(r1 , r0 ) = mcd(r0 , r1 ) = . . . = mcd(rn1 , rn ) = mcd(rn1 , 0) = rn1 .
11

Ejemplo 1.20. Veamos como el algoritmo de Euclides sirve no s


olo para calcular el
maximo com
un divisor de dos n
umeros, sino tambien para obtenerlo como combinaci
on
lineal de ellos, como indica el Teorema 1.8. por ejemplo, supongamos que queremos calcular
mcd(28, 72) y escribirlo en funci
on de 28 y 72. Efectuamos para ellos las sucesivas divisiones
eucldeas:
72 = 2 28 + 16
28 = 1 16 + 12
16 = 1 12 + 4
12 = 3 4 + 0.
Hemos llegado por tanto a un resto 0, luego el m
aximo com
un divisor de 72 y 28 es el
resto anterior, que es 4 (que es, por supuesto, el m
aximo com
un divisor obtenido a partir
de la descomposicion de 72, 28 en factores primos). Si vamos despejando este 4 desde la
pen
ultima divisi
on hacia arriba obtendremos:
4 = 16 1 12 = 16 1 (28 1 16) = 2 16 28 = 2(72 2 28) 28 = 2 72 5 28,
lo que nos da la expresi
on de 4 como combinacion de 72 y 28, conrmando por tanto el
Teorema de Bezout.
Teorema 1.21. Dados a, b, c Z y d = mcd(a, b), la ecuacion ax+by = c tiene soluci
on si
aneamente
y solo si d|c. Ademas, si x0 , y0 es una solucion de la ecuacion y , b no son simult
nulos, el conjunto de soluciones se puede escribir como

x = x0 + t
d
a

y = y0 t
d
con t Z.
Demostraci
on: Que la ecuacion tenga soluci
on si y s
olo si d|c (es decir, c es un m
ultiplo de
d) es una mera reformulaci
on del Teorema 1.8. Por otra parte, supongamos que tenemos
aneamente
una soluci
on ja x0 , y0 , es decir, c = ax0 +by0 . El hecho de que a, b no son simult
cero quiere decir que d = 0, por lo que podemos dividir por d. Es claro entonces que los
on de la ecuacion.
pares de la forma (x0 + db t, y0 ad t) son soluci
Recprocamente, si x, y es una solucion de la ecuacion, entonces ax + by = ax0 + by0 ,
que es equivalente a a(x x0 ) = b(y0 y) o tambien ad (x x0 ) = db (y0 y), por lo que
12

a b
d | d (y0

y). Por la Proposici


on 1.5(ii) se tiene mcd( ad , db ) = 1, luego por el Lema de
Euclides (Teorema 1.13) se tiene ad |y0 y, luego existe t Z tal que y0 y = ad t, es decir,
y = y0 ad t. Sustituyendo y por este valor en ad (x x0 ) = db (y0 y) se obtiene x = x0 + db t,
luego todas las soluciones son de la forma del enunciado.

Ejemplo 1.22. Retomemos el Ejemplo 1.20 para ver como resolver de forma pr
actica una
ecuacion como la del teorema anterior. Nos planteamos resolver la ecuacion 72x + 28y = 8.
Como mcd(72, 28) = 4 y 4|8, la ecuacion tiene soluci
on. Adem
as, vimos que podemos
escribir 4 = 2 72 5 28, por lo que multiplicando por 2 obtenemos que x0 = 4, y0 = 10
es una solucion. Las soluciones de la ecuacion seran, por tanto, de la forma
x=

4 + 7t

y = 10 18t
con t Z. Observese que este proceso puede interpretarse como encontrar todos los puntos
de la recta 72x+28y = 8 del plano afn que tienen sus dos coordenadas enteras. El Teorema
1.21 nos da una caracterizaci
on de cu
ando una recta afn dada por una ecuaci
on implcita
con coecientes enteros tiene puntos con coordenadas enteras. Ademas, en caso armativo,
los puntos de coordenadas enteras se pueden obtener dando valores enteros a una ecuaci
on
parametrica (con coecientes enteros) de la recta.

13

2. Teora de congruencias
Denici
on. Fijado un n
umero entero positivo n y dados n
umeros enteros a, b, diremos
que a es congruente con b m
odulo n, y lo denotaremos con a b (mod n), si n|a b.
Proposici
on 2.1. Fijado un entero n > 0 y dados n
umeros enteros a, b, c, d, se tiene:
(i) a a (mod n).
(ii) a b (mod n) si y solo si b a (mod n).
(iii) Si a b (mod n) y b c (mod n), entonces a c (mod n).
(iv) Si a b (mod n) y c d (mod n), entonces y a + c b + d (mod n).
(v) Si a b (mod n) y c d (mod n),, entonces ac bd (mod n).
(vi) Si ac bc (mod n), entonces a b (mod nd ), donde d = mcd(c, n). En particular, si
mcd(c, n) = 1, entonces ac bc (mod n) si y solo si a b (mod n).
Demostraci
on: Es un simple ejercicio que se deja al lector. Hagamos al menos la demostracion de la parte (vi). Por denici
on, ac bc (mod n) quiere decir n|acbc = (ab)c.
n
on 1.5(ii)),
Dividiendo por d tendremos d |(a b) dc . Como mcd( nd , dc ) = 1 (ver Proposici
n
por el Lema de Euclides (Teorema 1.13) se sigue d |(a b), es decir, a b (mod nd ).
Observaci
on 2.2. En lenguaje m
as matematico, observese que las tres primeras propiedades de la Proposici
on 2.1 estan diciendo que la relaci
on (mod n) es una relaci
on de
equivalencia ((i) es la propiedad reexiva, (ii) es la simetrica y (iii) es la transitiva). El

cociente por esta relacion es lo que en Algebra


B
asica se denota por Zn , y las propiedades
(iv) y (v) nos dicen que tal cociente tiene estructura de anillo. N
otese que, por la propiedad
(ii), el orden de los elementos en la congruencia es indistinto, por lo que se puede hablar
de que a y b son congruentes entre s o no.
Una pregunta natural es si todos los enteros son congruentes a n
umeros sencillos. El
siguiente resultado da una respuesta satisfactoria a dicha pregunta:
Proposici
on 2.3. Dado n un n
umero entero positivo, cada n
umero entero a es congruente
modulo n con uno y s
olo uno de los elementos del conjunto {0, 1, . . . , n 1}, precisamente
al resto de la divisi
on eucldea de a entre n. En particular, dos enteros a, b son congruentes
modulo n si y solo si a y b tienen el mismo resto al dividir entre n.
Demostraci
on: Sea a = qn + r la divisi
on eucldea de a entre n. Es claro que n|a r, por
lo que a r (mod n). Por otra parte, si a s (mod n) para alg
un s {0, 1, . . . , n 1},
entonces n|a s, luego se podra escribir a = cn + s, para alg
un c Z. Como 0 s < n,
14

la unicidad de la divisi
on eucldea (Teorema 1.6) implica que a = cn + s es precisamente
la divisi
on eucldea de a , por lo que s = r.
De hecho, la proposici
on anterior no hace sino decir en otro lenguaje un resultado
que ya conocamos: que, jado n, cada n
umero entero se puede escribir de una sola de
las formas nk + r, con r {0, 1, . . . , n 1}. Si cambiamos el conjunto {0, 1, . . . , n 1},
tendremos la siguiente denici
on.
Denici
on. Se llama sistema completo de restos m
odulo n a un conjunto R de n
umeros
enteros tales que cada a Z es congruente modulo n a un elemento de R y solo a uno.
Por ejemplo, {8, 3, 6, 1} sera un sistema completo de restos modulo 4, ya que
8 0 (mod 4), 3 1 (mod 4), 6 2 (mod 4) y 1 3 (mod 4). En otras palabras,
comparando con la Observaci
on 1.7, estamos diciendo que cada n
umero entero es de una
y solo una de las formas 4k + 8, 4k 3, 4k + 6, 4k 1. Una partici
on as puede parecer
arbitraria, pero por ejemplo el lector puede darse cuenta ahora de que nuestra demostraci
on
de la existencia de innitos n
umeros primos de la forma 4k + 3 utilizaba en realidad los
n
umeros de la forma 4k 1.
Damos a continuaci
on un metodo u
til para caracterizar sistemas completos de restos
modulo un n
umero n (que da en particular, como condici
on necesaria, que deben ser
conjuntos de n elementos).
Lema 2.4. Sea n un n
umero entero positivo y sea R = {r1 , r2 , . . . , rm } un conjunto de
n
umeros enteros. Sean r1 , r2 , . . . , rm respectivamente los restos de la division eucldea entre
n de r1 , r2 , . . . , rm . Entonces son equivalentes:
(i) R es un sistema completo de restos modulo n.
(ii) m n y para cada i = j se tiene ri rj (mod n).
(iii) Los n
umeros r1 , r2 , . . . , rm coinciden, en cierto orden, con 0, 1, . . . , n 1.
Demostraci
on: Haremos la demostracion de forma cclica.
(i)(ii): Como cada uno de los n
umeros 0, 1, . . . , n 1 son congruentes modulo n a
alg
un elemento de R y dos de ellos no son congruentes entre s, necesariamente R debe
contener al menos n elementos, es decir, m n.
umero ri sera
Ademas, si para alg
un i = j fuera ri rj (mod n), entonces el n
simult
aneamente congruente modulo n con ri y rj , contrario a la denici
on de sistema
completo.
(ii)(iii): Si i = j, tenemos por hip
otesis ri rj (mod n), luego, por la Proposici
on
2.3, se tendr
a ri = rj . Por tanto, los n
umeros r1 , r2 , . . . , rm son m n
umeros distintos entre
los n n
umeros 0, 1, . . . , n 1. Como m n, ambos sistemas de n
umeros deben coincidir.
15

(iii)(i): Por la Proposici


on 2.3, cada n
umero entero es congruente con uno y s
olo uno
de los n
umeros 0, 1, . . . , n 1, que por hip
otesis son los n
umeros r1 , r2 , . . . , rm . Como, de
nuevo por la Proposici
on 2.3, un n
umero es congruente con ri si y s
olo si es congruente con
umero entero es congruente con uno y s
olo uno de los n
umeros
ri , se concluye que cada n
r1 , r2 , . . . , rm , es decir, R es un sistema completo de restos modulo n.
A partir del lema, podemos ya dar muchas formas de construir sistemas completos de
restos modulo n:
Proposici
on 2.5. Dado un entero positivo n, los siguientes conjuntos son sistemas completos de restos modulo n:
(i) Cualquier conjunto de n enteros consecutivos.
(ii) {ar1 + b, ar2 + b, . . . , arn + b}, donde {r1 , r2 , . . . , rn } es un sistema completo de restos
modulo n, a es un entero tal que mcd(a, n) = 1 y b es un entero cualquiera.
(iii) {b, a + b, 2a + b, . . . , (n 1)a + b}, donde a es un entero tal que mcd(a, n) = 1 y b es
un entero cualquiera.
Demostraci
on: Es todo consecuencia del Lema 2.4(ii). Para el conjunto (i) usamos que
tiene n elementos y que claramente no hay dos elementos que sean congruentes entre
s. Para el conjunto (ii), usamos que, por ser {r1 , r2 , . . . , rn } es un sistema completo de
restos modulo n, si i = j entonces ri rj (mod n). Como mcd(a, n) = 1, se sigue
de la Proposici
on 2.1 que ari arj (mod n) y ari + b arj + b (mod n). El Lema
2.4(ii) implica entonces que {ar1 + b, ar2 + b, . . . , arn + b} es un sistema completo de restos
modulo n. Finalmente, (iii) es el caso particular de (ii) en que tomamos {r1 , r2 , . . . , rn } =
{0, 1, . . . , n 1}.
Ejemplo 2.6. Se pueden usar las congruencias para dar criterios de divisibilidad. Recordemos que un n
umero con expresi
on decimal ar ar1 . . . a1 a0 quiere decir el n
umero n =
r
r1
+ . . . + a1 10 + a0 . Entonces, por ejemplo:
ar 10 + ar1 10
Como 10 1 (mod 3), se sigue de las propiedades del Lema 2.1 que n ar +
ar1 + . . . + a1 + a0 (mod 3). En particular se obtiene el resultado conocido: un n
umero
es divisible por 3 si y s
olo si la suma de sus cifras es divisible por 3.
Como tambien 10 1 (mod 9), se obtiene del mismo modo que un n
umero es
divisible por 9 si y solo si la suma de sus cifras es divisible por 9. M
as a
un, de la congruencia
n ar + ar1 + . . . + a1 + a0 (mod 3) se sigue la clasica prueba del nueve. Por ejemplo,
si queremos multiplicar 53914 por 13518, usamos que
161277 1 + 6 + 1 + 2 + 7 + 7 = 24 2 + 4 = 6 (mod 9)
16

140263 1 + 4 + 0 + 2 + 6 + 3 = 16 1 + 6 = 7 (mod 9)
por lo que debe ser
161277 140263 6 7 = 42 4 + 2 = 6 (mod 9).
Por tanto, una forma de comprobar que el producto est
a bien calculado es comprobar del
mismo modo que es congruente con 6 modulo 9. Y, en efecto, tenemos:
22621195851 2 + 2 + 6 + 2 + 1 + 1 + 9 + 5 + 8 + 5 + 1 = 42 4 + 2 = 6 (mod 9).
Obviamente, si la prueba no sale es que la operaci
on esta mal hecha, pero el que salga bien
la prueba no implica que la operaci
on este bien hecha (por ejemplo, 22531195851 hubiera
dado el mismo resultado m
odulo 9).
Si usamos ahora 10 1 (mod 11), obtenemos el criterio que un n
umero es divisible
por 11 si y solo si la suma con signo alternado de sus cifras es divisible por 11, ya que
n (1)r ar + (1)r1 ar1 + . . . a1 + a0 (mod 11). De hecho, este hecho nos permite
inventarnos una prueba del once. Siguiendo con el ejmplo anterior, tendremos:
161277 1 + 6 1 + 2 7 + 7 = 6 (mod 11)
140263 1 + 4 0 + 2 6 + 3 = 2 (mod 11)
por lo que debe ser
161277 140263 6 2 = 12 1 + 2 = 1 (mod 11).
En efecto, tenemos
22621195851 2 2 + 6 2 + 1 1 + 9 5 + 8 5 + 1 = 12 1 + 2 = 1 (mod 11).
Observese que esta nueva prueba servira para reforzar la prueba del nueve, ya que ahora
22531195851 2 2 + 5 3 + 1 1 + 9 5 + 8 5 + 1 = 10 1 (mod 11), lo que indica
que 22531195851 no puede ser el producto.
Muchas veces es mas sencillo reducir una congruencia m
odulo un entero n a congruencias con modulos m
as peque
nos que n. El siguiente resultado explica c
omo:
umeros naturales tales que mcd(ni , nj ) = 1 si
Proposici
on 2.7. Sean n1 , n2 , . . . , nr n
i = j. Entonces, para cada par de enteros a, b se tiene que a b (mod n1 n2 . . . nr ) si y
solo si a b (mod n1 ), a b (mod n2 ), . . . , a b (mod nr ).
Demostraci
on: Por denici
on, a b (mod n1 n2 . . . nr ) si y solo si n1 n2 . . . nr |a b. Como
mcd(ni , nj ) = 1 si i = j, el Corolario 1.12 implica que n1 n2 . . . nr |a b es equivalente a
que, para cada i = 1, 2, . . . , r, se tenga ni |a b, es decir, a b (mod ni ).
Si en cambio queremos pasar de una congruencia m
odulo n a congruencias modulo un
u
nico divisor de n, entonces el resultado es mas complicado:
17

Proposici
on 2.8. Si d|n entonces a b (mod nd ) si y solo si a b + i nd (mod n) para
alg
un i {0, 1, . . . , d 1}. Ademas, los n
umeros b, b + nd , . . . , b + (d 1) nd son dos a dos
incongruentes m
odulo n.
Demostraci
on: Por una parte, es claro que, si a b + i nd (mod n), entonces existe c Z
tal que a b i nd = cn. Por tanto, a = b + nd (i + cd), luego a b (mod nd ).
Recprocamente, si a b (mod
divisi
on eucldea de t entre d:

n
d ),

existe t Z tal que a = b + nd t. Consideramos la

t = qd + r
con 0 r < d. Por tanto, podremos escribir
a=b+

n
n
n
t = b + (qd + r) = b + nq + r .
d
d
d

De aqu se deduce a b + r nd (mod n) y, como 0 r < d, es una de las d congruencias


que queramos.
odulo
Finalmente, los n
umeros b, b + nd , . . . , b + (d 1) nd son dos a dos incongruentes m
n porque son un subconjunto de los n
umeros b, b + 1, . . . , b + (n 1), que es un sistema
completo de restos modulo n (Proposici
on 2.5(i)).
Con el lenguaje de congruencias, podemos reinterpretar el Teorema 1.21 del siguiente
modo:
Proposici
on 2.9. Sean a, b, n numeros enteros con n > 0 y a = 0, y sea d = mcd(a, n).
Entonces la congruencia ax b (mod n) tiene solucion si y s
olo si d|b. Ademas, en este
caso hay exactamente d soluciones modulo n.
Demostraci
on: La congruencia tiene soluci
on si y s
olo si existe x Z tal que n|b ax, es
decir, si y solo si existen x, y Z tales que b ax = ny. Como el Teorema 1.21 arma que
la ecuacion ax + ny = b tiene solucion si y s
olo si d|b, la primera parte queda demostrada.
Ademas, el mismo Teorema 1.21 dice que las soluciones de la ecuacion dependen de un
par
ametro t y son de la forma

n
x = x0 + t
d
a
y = y t.
0
d
Entonces las soluciones de la congruencia son los enteros x de la forma x x0 (mod nd )
que, por la Proposici
on 2.8, da d soluciones modulo n.
Ejemplo 2.10. Consideremos la congruencia lineal 72x 8 (mod 28). Como indica la
demostracion de la Proposici
on 2.9, para resolverla, debemos resolver primero la ecuaci
on
18

72x + 28y = 8, que ya resolvimos en el Ejemplo 1.22, y cuya soluci


on era
x=

4 + 7t

y = 10 18t.
Por tanto, la soluci
on a nuestra congruencia es x = 4+7t, es decir x 4 (mod 7) o, escrito
modulo 28, x 4, 11, 18, 25 (mod 28).
La Proposici
on 2.9 nos dice que toda congruencia de la forma ax b (mod n) es equivalente a varias congruencias de la forma x x0 (mod n) (o en realidad a una congruencia
de la forma x x0 (mod nd )). Por tanto, si queremos resolver varias congruencias lineales
al mismo tiempo basta resolver un sistema de congruencias todas del tipo x x0 (mod n).
El siguiente teorema nos dice como resolver tales sistemas. El interes del resultado no sera
solo el enunciado te
orico (existencia de solucion bajo ciertas hip
otesis), sino tambien la
demostracion, que da un metodo efectivo de resolucion.
Teorema 2.11 (chino del resto). Sean n1 , n2 , . . . , nr n
umeros enteros positivos tales que
mcd(ni , nj ) = 1 si i = j. Entonces cualquier sistema de congruencias

x a1 (mod n1 )

x a2 (mod n2 )
..

x ar (mod nr )
tiene solucion, que adem
as es u
nica m
odulo n1 n2 . . . nr .
Demostraci
on: Para cada i = 1, 2, . . . , r denimos
Ni =

n1 n2 . . . nr
= n1 n2 . . . ni1 ni+1 . . . nr .
ni

Por el Corolario 1.10 se tendr


a mcd(ni , Ni ) = 1, ya que mcd(ni , nj ) = 1 para cada factor
on 2.9, existe xi Z tal que Ni xi ai (mod ni ), ya
nj de Ni . Por tanto, por la Proposici
que obviamente mcd(ni , Ni )|ai . Veamos que
x0 = N1 x1 + N2 x2 + . . . + Nr xr
es una soluci
on del sistema de congruencias. En efecto, para cada i = 1, 2, . . . , r se tiene
Nj 0 (mod ni ) si i = j, luego x0 Ni xi ai (mod ni ).
Para la unicidad, observemos en primer lugar que, si x x0 (mod n1 n2 . . . nr ), es
claro que x es tambien solucion del sistema de congruencias. Recprocamente, sea x una
19

soluci
on del sistema. Entonces x x0 (mod ni ) para cada i = 1, 2, . . . , r, luego, por la
Proposici
on 2.7, x x0 (mod n1 n2 . . . nr ).
Ejemplo 2.12. Veamos que el resultado anterior se puede usar incluso si los ni no
son todos primos entre s (con la diferencia de que entonces puede ocurrir que el sistema
de congruencias no tenga soluci
on). Supongamos por ejemplo que queremos resolver el
sistema de congruencias

x 5 (mod 24)

x 1 (mod 28)

x 4 (mod 15)
Evidentemente, no estamos en las condiciones del Teorema Chino del Resto, ya que, por
ejemplo, mcd(24, 28) = 4. Sin embargo, podemos usar la Proposici
on 2.7 para sustituir
nuestro sistema por el sistema equivalente:

x 5 (mod 8)

x 5 (mod 3)

x 1 (mod 4)

x 1 (mod 7)

x 4 (mod 3)

x 4 (mod 5).
Por supuesto, seguimos sin estar en las hip
otesis del Teorema Chino del Resto, pero ahora
vamos a poder quitar las congruencias que nos sobran. En primer lugar, tenemos las congruencias x 5 (mod 3) y x 4 (mod 3), que son equivalentes, ya que 5 4 (mod 3),
as que podemos eliminar una de ellas. A
un tenemos el par de congruencias x 5 (mod 8)
y x 1 (mod 4) que no nos permiten aplicar el Teorema Chino del Resto. En este caso,
observamos que x 5 (mod 8) implica obviamente x 5 (mod 4), que es equivalente
a x 1 (mod 4), porque 5 1 (mod 4). As pues, podemos eliminar la congruencia
x 1 (mod 4) y tenemos ya el sistema equivalente de congruencias:

x 5 (mod 8)

x 5 (mod 3)

x 1 (mod 7)

x 4 (mod 5)
que ya esta en las hip
otesis del Teorema Chino del Resto, luego ya sabemos que tiene
solucion (vale la pena hacer aqu una pausa para observar que podra haber ocurrido que,
20

en lugar de encontrar congruencias equivalentes m


odulo 4, hubieramos encontrado congruencias incompatibles; esto hubiera querido decir que el sistema de partida es incompatible y
no tiene soluci
on). Aplicamos ahora el metodo de resolucion que nos da el Teorema Chino
del Resto, es decir, resolvemos primero (con el metodo explicado en el Ejemplo 2.10) las
congruencias:
105x 5(mod 8), que tiene como solucion x 5(mod 8).
280x 5(mod 3), que tiene como solucion x 2 (mod 3).
120x 1(mod 7), que tiene como solucion x 1 (mod 7).
168x 4(mod 5), que tiene como solucion x 2 (mod 5).
La solucion nal queda, por tanto
x 105 5 + 280 2 + 120 1 + 168 2 = 1541 701 (mod 840).
Como nos indica el ejemplo anterior, para estudiar congruencias, basta estudiar congruencias modulo la potencia de un primo. El caso m
as simple es el de congruencias modulo
un primo, sobre el que se pueden demostrar toda una serie de resultados. El primero de
todos es el siguiente:
Teorema 2.13 (Fermat). Sea p un n
umero primo. Entonces, para cada a Z no divisible
p1
por p se tiene a
1 (mod p). Como consecuencia, ap a (mod p) para cualquier
a Z.
Demostraci
on: Por la Proposici
on 2.5(iii), sabemos que, si mcd(a, p) = 1, los n
umeros
0, a, 2a, . . . , (p 1)a forman un sistema completo de restos modulo p. Por el Lema 2.4, esto
quiere decir que sus restos al dividir por p son, en cierto orden, 0, 1, 2, . . . , p 1. Como
obviamente el cero del primer conjunto se corresponde con el cero del segundo, se tiene que
los n
umeros a, 2a, . . . , (p1)a son congruentes modulo p, en cierto orden, con 1, 2, . . . , p1.
Multiplicando todas las congruencias tendremos (p 1)!ap1 (p 1)! (mod p). Ademas,
como p es un n
umero primo, es coprimo con todos los factores 1, 2, . . . , p 1 de (p 1)!,
luego (Corolario 1.10) mcd((p 1)!, p) = 1. Se sigue entonces de la Proposici
on 2.1(vi) que
p1
1 (mod p).
a
La congruencia anterior implica autom
aticamente ap a (mod p), siempre bajo la
hip
otesis mcd(a, p) = 1. Si en cambio p|a, entonces tanto ap como a son congruentes con
alida en este caso.
cero modulo p, luego la congruencia ap a (mod p) es tambien v
La propiedad del Teorema de Fermat no es caracterstica de los n
umeros primos, ya
que hay n
umeros compuestos que satisfacen lo mismo.
21

Denici
on. Se llama n
umero pseudoprimo en base a a un n
umero compuesto n tal que
umero pseudoprimo a un n
umero compuesto n tal
an a (mod n). Se llama simplemente n
n
que 2 2 (mod n). Finalmente, se llama pseudoprimo absoluto o n
umero de Carmichael
n
umero entero a.
a un n
umero compuesto n tal que a a (mod n) para cualquier n
Obviamente son mucho m
as interesantes (y mas difciles de encontrar) los n
umeros de
Carmichael que los pseudoprimos (aunque los matem
aticos chinos de hace 25 siglos pensaban que no haba n
umeros pseudoprimos). De ambos se sabe que hay innitos de ellos. En
el caso de pseudoprimos, no cuesta mucho ver que, dado un n
umero pseudoprimo impar
n
umero pseudoprimo, lo que permite construir innitos
n, entonces 2 1 es tambien un n
pseudoprimos (basta empezar por 341 = 11 31, que es el primer n
umero pseudoprimo).
Para n
umeros de Carmichael, nos limitaremos a dar el siguiente resultado:
Proposici
on 2.14. Sea n = p1 p2 . . . pr un producto de r primos distintos, con r > 1. Si
para cada i = 1, 2, . . . , r se tiene pi 1|n 1, entonces n es un n
umero de Carmichael.
Demostraci
on: Claramente n es compuesto, porque r > 1, as que hay que demostrar que
n
a a (mod p1 p2 . . . pr ) para todo entero a. Por la Proposici
on 2.7, hay que demostrar
n
que, para cada i = 1, 2, . . . , r, se tiene a a(mod pi ). Obviamente esto es cierto si pi |a,
as que podemos suponer mcd(a, pi ) = 1. En tal caso, por el Teorema de Fermat se tendra
otesis pi 1|n1, se tendr
a tambien an1 1(mod pi ),
api 1 1 (mod pi ) y, como por hip
lo que implica an a(mod pi ).
Ejemplo 2.15. Con la proposici
on anterior, se demuestra que 561 = 3 11 17 es un
n
umero de Carmichael (de hecho es el primero de todos). Otros ejemplos obtenidos de este
modo son los n
umeros 1729 = 7 13 19, 6601 = 7 23 41 y 10585 = 5 29 73. Aunque
no veriquen el critero de la proposici
on anterior, tambien son n
umeros de Carmichael
1105 = 5 13 17, 2821 = 7 13 31, 15841 = 7 31 73 y 16046641 = 13 37 73 457 (uno
de los pocos conocidos con cuatro factores, descubierto por el propio Carmichael). S
olo en
1994 se demostro que existen innitos n
umeros de Carmichael. Son tan escasos que solo
hay 43 de ellos menores que un mill
on.
Ejemplo 2.16. Si consideramos el conjunto de restos m
odulo 11, observamos que tenemos
los pares disjuntos {2, 6}, {3, 4}, {5, 9}, {7, 8} con la propiedad de que el producto de los
elementos de un par es congruente con 1 modulo 11. Adem
as, la uni
on de todos los pares
da el conjunto {2, 3, 4, 5, 6, 7, 8, 9}, por lo que tendremos
2 3 4 5 6 7 8 9 = (2 6)(3 4)(5 9)(7 8) 1 (mod 11).
Esto no es casualidad, y para cada n
umero primo tenemos la misma situacion, lo que nos
permite demostrar el siguiente teorema.
22

Teorema 2.17 (Wilson). Si p es un n


umero primo, entonces (p 1)! 1 (mod p).
Demostraci
on: Suponemos p > 3, ya que los casos p = 2, 3 se pueden comprobar trivialmente a mano. Para cada a = 1, 2 . . . , p 1, consideramos la congruencia ax 1 (mod p)
que, por la Proposici
on 2.9 tendr
a exactamente una solucion m
odulo p, luego se podr
a
representar exactamente por un n
umero a {1, 2, . . . , p 1} (claramente, no puede ser
a = 0). Por tanto, tenemos una aplicaci
on {1, 2, . . . , p 1} {1, 2, . . . , p 1} que
on es necesariamente biyectiva, al tener
asocia a cada a el elemento a . Tal aplicaci
ambos conjuntos el mismo n
umero de elementos. Observese que, si a = a, entonces
2
2
a 1 (mod p), es decir p|a 1 = (a + 1)(a 1). Por el Corolario 1.14, se tiene p|a + 1
o bien p|a 1. Como a {1, 2, . . . , p 1}, se sigue a = 1 o bien a = p 1. Por tanto,
el conjunto {2, 3, . . . , p 2} se descompone, como en el Ejemplo 2.16, en pares {a, a }
un estos pares el producto de todos
con la propiedad aa 1 (mod p). Agrupando seg
los elementos de {2, 3, . . . , p 2}, se llega a 2 3 . . . (p 2) 1 (mod p) y, por tanto,
(p 1)! = 1 (2 3 . . . (p 2))(p 1) 1 1 (p 1) 1 (mod p), como queramos.

Observaci
on 2.18. Notese que, al contrario que pasa con el teorema de Fermat, la
propiedad anterior s que caracteriza a los n
umeros primos. En efecto, dado un n
umero
compuesto n, si d = 1, n es un divisor suyo, se tendr
a que d es uno de los factores de
(n 1)!, por lo que (n 1)! 0 (mod d). Por tanto, no puede ser (n 1)! 1 (mod n).
En realidad, no es difcil demostrar que, si n > 4 es un n
umero compuesto, entonces
(n 1)! 0 (mod n).
El Teorema de Wilson tiene el siguiente corolario inesperado:
Teorema 2.19. Si p es primo, la congruencia x2 1 (mod p) tiene solucion si y s
olo si
p = 2 o bien p 1 (mod 4).
Demostraci
on: Supongamos primero que la congruencia tiene una soluci
on, que llamaremos
2
a. Obviamente, a no puede ser divisible por p, porque en tal caso a 0 (mod p). Para
ver que p = 2 o bien p 1 (mod 4) basta demostrar que p 3 (mod 4). En efecto, si
sera un n
umero impar, y como a2 1 (mod p),
fuera p 3 (mod 4), entonces p1
2
entonces
ap1 = (a2 )

p1
2

(1)

p1
2

(mod p) = 1 (mod p),

mientras que el Teorema de Fermat (Teorema 2.13), implica ap1 1 (mod p), lo que da
una contradicci
on.
Recprocamente, supongamos p = 2 o bien p 1 (mod 4) y veamos que la congruencia
tiene solucion. Es claro que x2 1 (mod 2) tiene soluci
on (de hecho, cualquier x impar
23

lo es), as que podemos limitarnos al caso p 1 (mod 4). En tal caso, tenemos que
es un n
umero par. Esto quiere decir que el n
umero de congruencias (triviales)

p1
2

1 (p 1) (mod p)
2 (p 2) (mod p)
..
.
p1
p+1

(mod p)
2
2
es par. Por tanto, multiplic
andolas todas tendremos
p+1
p1
)! (p 1)(p 2) . . .
(mod p)
2
2
de donde se deduce, usando el Teorema de Wilson para la u
ltima congruencia,
(

p1
)!
2

(p 1)(p 2) . . .

p+1 p1
(
)! = (p 1)! 1 (mod p).
2
2

on de x2 1 (mod p).
Por tanto, x = ( p1
2 )! es una soluci

Corolario 2.20. Existen innitos n


umeros primos de la forma 4k + 1.
Demostraci
on: Como siempre, supongamos por reducci
on al absurdo que p1 , p2 , . . . , pr
sean todos los n
umeros primos de la forma 4k + 1. Consideramos entonces el n
umero
2
a = (2p1 p2 . . . pr ) + 1 y sea p un divisor primo de a (que obviamente existe, por ser
a > 4. Entonces, 2p1 p2 . . . pr es una solucion de la congruencia x2 1 (mod p), por lo
que el Teorema 2.19 implica que p = 2 o bien p es de la forma 4k + 1, es decir, es uno
de los p1 , p2 , . . . , pr . Como claramente ni 2 ni p1 , p2 , . . . , pr son divisores de a, se llega la
contradicci
on buscada.
Podemos mejorar el resultado anterior de la misma forma que hicimos en la Observacion 1.7 con el Teorema de Euclides:
Corolario 2.21. Existen innitos n
umeros primos de la forma 8k + 5.
umeros primos de la forma
Demostraci
on: Supongamos que p1 , p2 , . . . , pr sean todos los n
2
8k + 5, y consideramos de nuevo a = (2p1 p2 . . . pr ) + 1. Ya hemos observado en la
demostracion anterior que los divisores primos de a son de la forma 4k + 1. Por tanto, son
de la forma 8k + 1 o bien 8k + 5. Ahora bien, como cada (p1 p2 . . . pr )2 es impar, es decir,
de la forma 2k + 1, entonces a = 4(p1 p2 . . . pr )2 + 1 es de la forma 8k + 5. Por tanto, no
todos los divisores primos de a son de la forma 8k + 1, por lo que a es divisible por alg
un
primo de la forma 8k + 5, es decir, por alg
un pi , lo que es imposible.
24

3. Funciones aritmeticas
Vamos a intentar extender ahora el Teorema de Fermat (Teorema 2.13) cuando el
exponente es un n
umero arbitrario n no necesariamente primo. Mirando a la demostraci
on,
vamos a necesitar multiplicar solo los restos modulo n que sean coprimos con n (pues si
no, no podramos cancelarlos luego). Ahora bien, cu
antos restos hay coprimos con n?
Claramente, esto dependera de n, es decir, sera una funci
on, a la que de momento damos
un nombre (nuestro objetivo en este captulo ser
a estudiar funciones de este tipo):
Denici
on. Se llama funci
on de Euler a la funci
on : Z1 Z que asocia a cada
entero n 1 el n
umero (n) de enteros r {1, 2, . . . , n} tales que mcd(r, n) = 1. N
otese
que, como ni r = 0 ni r = n verican mcd(r, n) = 1, se puede denir tambien (n) como
el n
umero de enteros r {0, 1, . . . , n 1} tales que mcd(r, n) = 1.
Con esta denici
on, la generalizaci
on del Lema 2.4 (en realidad s
olo la implicaci
on
que nos intereresa) es la siguiente:
umeros enteros enteros
Lema 3.1. Sea n un n
umero entero positivo y sean r1 , r2 , . . . , rm n
tales que, m (n), para todo i = 1, 2, . . . , m se tiene mcd(ri , n) = 1 y mcd(ri , rj ) = 1
si j = i. Entonces, los restos de la division eucldea de r1 , r2 , . . . , rm entre n son todos
distintos y coinciden, en cierto orden, con los n
umeros entre 0 y n que son coprimos con
n.
Demostraci
on: Consideramos en primer lugar los n
umeros s1 , s2 , . . . , sr {0, 1, . . . , n 1}
on, r = n
tales que mcd(sj , n) = 1 para j = 1, 2, . . . , r. Observese que, por denici
(n). Como s1 , s2 , . . . , sr forman parte de un sistema completo de restos modulo n, son
un
incongruentes dos a dos m
odulo n. Ademas, ning
un ri es congruente modulo n con ning
otesis. Por
sj , ya que eso querra decir que mcd(ri , n) = mcd(sj , n), contra nuestra hip
tanto, por el Lema 2.4, como m + r = m + n (m) n, se tiene que los restos de la
divisi
on de r1 , r2 , . . . , rm , s1 , s2 , . . . , sr entre n son todos distintos y coinciden, en cierto
orden con 0, 1, . . . , n 1. Como los restos de s1 , s2 , . . . , sr son ellos mismos, el resultado
queda demostrado.
Con estas herramientas, podemos proceder ya a generalizar el Teorema de Fermat:
Teorema 3.2 (Euler). Dado un n
umero entero n 2, entonces para cada n
umero entero
(n)
1 (mod n).
a tal que mcd(a, n) = 1 se tiene a
Demostraci
on: Sean r1 , . . . , r(n) los enteros positivos menores que n tales que mcd(ri , n) =
1. Como mcd(a, n) = 1, la Proposici
on 1.9 implica que mcd(ari , n) = 1 para todo i =
1, 2, . . . , (n) y, si j = i, la Proposici
on 2.1(vi) implica ari arj (mod n). Por el Lema
25

3.1, los n
umeros ar1 , . . . , ar(n) son congruentes, en alg
un orden, con r1 , . . . , r(n) modulo
n. Por tanto, haciendo el producto de todos ellos tendremos
a(n) r1 . . . r(n) r1 . . . r(n) (mod n).
on 1.9), la Proposici
on 2.1(vi) implica
Como mcd(r1 . . . r(n) , n) = 1 (por la Proposici
(n)
a
1 (mod n).
Como consecuencia, obtenemos un metodo pr
actico para resolver una congruencia
lineal:
Corolario 3.3. Si mcd(a, n) = 1, la congruencia lineal ax b (mod n) es equivalente a
x a(n)1 b (mod n).
Demostraci
on: Por la Proposici
on 2.9), la congruencia tiene una u
nica soluci
on, as que
(n)1
(n)1
b es una soluci
on. Pero esto es inmediato, ya que a(a
b) = a(n) b
basta ver que a
es, por el Teorema de Euler, congruente con b modulo n.

Observaci
on 3.4. Claramente, el Teorema de Euler es una generalizacion del peque
no
teorema de Fermat, ya que si p es un n
umero primo es claro que (p) = p 1 (puesto
que 1, 2, . . . , p 1 son los n
umeros coprimos con p y menores que el). M
as en general, si
k
umero primo, es claro que los enteros positivos menores o iguales que n
n = p , con p n
que no son primos con n son los m
ultiplos de p, es decir, 1 p, 2 p, . . . , pk1 p. Como hay
pk1 de ellos, se sigue entonces que
1
(pk ) = pk pk1 = pk1 (p 1) = pk (1 ).
p
Calcular (n) para un n arbitrario no parece a priori f
acil. Sin embargo, tenemos la
siguiente propiedad que resuelve el problema.
Teorema 3.5. Si mcd(m, n) = 1, entonces (mn) = (m)(n).
Demostraci
on: Escribimos los primeros mn n
umeros en forma de rectangulo n m:
1
m+1
..
.

2
m+2
..
.

...
...

i
m+i
..
.

...
...

jm + 1
..
.

jm + 2
..
.

...

jm + i
..
.

...

(n 1)m + 1

(n 1)m + 2 . . . (n 1)m + i
26

...

m1
2m 1
..
.

m
2m
..
.

jm + m i (j + 1)m
..
..
.
.
mn 1
mn

es decir, escribimos cada n


umero como jm + i, con i = 1, 2, . . . , m y j = 0, 1, . . . , n 1. Por
la Proposici
on 1.9, un n
umero es primo con mn si y solo si es primo con m y con n, luego
(mn) sera el n
umero de elementos jm + i que son primos con m y con n. La primera
observacion es que (por la Proposici
on 1.5(i)) mcd(jm + i, m) = mcd(i, m), luego el que
un n
umero jm + i sea primo con m depende s
olo de la columna i-esima en que esta. Por la
propia denici
on de , tendremos exactamente (m) valores de i tales que mcd(i, m) = 1,
es decir, que los elementos de la tabla que son primos con m son los elementos de (m)
columnas.
Queda entonces ver cu
antos elementos en cada columna de las (n) anteriores son
ademas primos con n. De hecho, el teorema quedar
a demostrado si vemos que cada una
de esas columnas tiene exactamente (n) elementos primos con n. De hecho, cualquier
columna de la matriz tiene exactamente (n) elementos primos con n. En efecto, los elementos de una columna son de la forma i, m + i, 2m + i, . . . (n 1)m + i que, por la
Proposici
on 2.5(iii), forman un sistema completo de restos m
odulo n. Por tanto, son congruentes, aunque posiblemente en otro orden, a 0, 1, 2, . . . , n1, por lo que hay exactamente
(n) que sean coprimos con n.
Corolario 3.6. Si n = pk11 pk22 . . . pkr r es la descomposicion de n en factores primos, entonces (n) = pk11 1 pk22 1 . . . pkr r 1 (p1 1)(p2 1) . . . (pr 1) = n(1 p11 )(1 p12 ) . . . (1 p1r ).
Demostraci
on: Es consecuencia inmediata de que, aplicando reiteradamente el Teorema
3.5, se tiene (pk11 pk22 . . . pkr r ) = (pk11 )(pk22 ) . . . (pkr r ), y por otra parte, como vimos en la
Observaci
on 3.4, para cada i = 1, 2, . . . , r se tiene (pki i ) = pki i 1 (pi 1).
Ejemplo 3.7. Aunque el teorema de Euler sea una generalizaci
on del de Fermat, normalmente con este u
ltimo se puedun obtener resultados mejores. Por ejemplo, para estudiar
cuando una potencia es congruente con 1 m
odulo 35, el teorema de Euler dira que, como
(35) = (5 7) = (5)(7) = (5 1)(7 1) = 24,
entonces a24 1 (mod 35) si mcd(a, 35) = 1. Sin embargo, como mcd(a, 35) = 1 es
equivalente a mcd(a, 5) = 1 = mcd(a, 7) (ver Proposici
on 1.9), por el teorema de Fermat
se tiene
a4 1 (mod 5)
y
a6 1 (mod 7).
Elevando respectivamente al cubo y al cuadrado se tendr
a
a12 1 (mod 5)
27

y
a12 1 (mod 7),
de donde se deduce a12 1 (mod 35).
Denici
on. Una funci
on aritmetica es una funci
on f : Z1 Z. Se dice ademas que es
multiplicativa si f (mn) = f (m)f (n) cuando mcd(m, n) = 1.
Ejemplo 3.8.
plicativas:

De forma trivial se pueden construir varias funciones aritmeticas multi-

1) La funci
on identicamente cero es claramente multiplicativa.
2) Si una funci
on multiplicativa f no es identicamente cero, existe n Z1 tal que
f (n) = 0. Como se tiene f (n) = f (n 1) = f (n)f (1), se sigue entonces f (1) = 1. Esto
sugiere otro segundo ejemplo trivial de funci
on aritmetica multiplicativa:
f (n) =

1
0

si n = 1
si n > 1

3) De la misma forma, la funci


on identicamente uno es claramente multiplicativa.
4) La funci
on identidad id(n) = n es tambien claramente multiplicativa.
5) M
as en general, aunque no vayamos a usarla, para cada k 0, la funci
on f (n) = nk
es multiplicativa.
Todos los ejemplos anteriores son mucho mas multiplicativos que la funci
on , ya
que f (mn) = f (m)f (n) independientemente de que m y n tengan factores en com
un o no.
El siguiente resultado nos dar
a un modo de calcular funciones multiplicativas m
as serias
a partir de otras funciones multiplicativas, aunque sean triviales:
Teorema 3.9. Si f es una funci
on aritmetica multiplicativa, entonces la funci
on
F (n) =

f (d)
d|n

es tambien una funci


on multiplicativa.
Demostraci
on: Sean m, n Z1 tales que mcd(m, n) = 1. Por el Corolario 1.17(i), cada
divisor positivo de mn se puede escribir como dd con d|m y d |n. Podemos entonces
escribir
f (dd ) =

F (mn) =
d|m,d |n

f (d)f (d ) =
d|m,d |n

f (d)
d|m

28

f (d ) = F (m)F (n).
d |n

Ejemplo 3.10. Apliquemos la construcci


on anterior a los ejemplos que tenemos de
funciones multiplicativas.
1) Evidentemente, si f es identicamente cero, entonces F tambien lo es.
2) Si f vale cero para todo n, excepto para f (1) = 1, es claro que entonces F es
constantemente 1.
3) Cuando f es la funci
on constante 1, se suele escribir para la funci
on F correspondiente, y se tiene que (n) es el n
umero de divisores positivos de n.
4) Cuando f = id, la funci
on F se suele denotar , y se tiene que (n) es la suma de
los divisores positivos de n.
5) Cuando f = , en principio no es f
acil deducir quien es F . La ventaja es que
sabemos que F es multiplicativa, por lo que basta calcularla para potencias de primos.
Usando que los divisores de una potencia de un primo p son las potencias menores de p,
tendremos, por la Observaci
on 3.4:
F (pk ) = (1) + (p) + (p2 ) + . . . + (pk1 ) + (pk ) =
= 1 + (p 1) + (p2 p) + . . . + (pk1 pk2 ) + (pk pk1 ) = pk .
Por tanto, F es la identidad en potencias de primos, y por ser multiplicativa se tendr
a
F = id. En otras palabras, hemos demostrado que para todo n Z1 se verica:
(d) = n
d|n

Observaci
on 3.11. El hecho de que las funciones y anteriores son multiplicativas se puede deducir directamente de su propia denici
on. En efecto, dado un n
umero
k1 k2
on 1.16)
p1 p2 . . . pkr r , con p1 , . . . , pr primos distintos, sus divisores son (por la Proposici
a1
ar
los n
umeros de la forma p1 . . . pr con 0 ai ki , es decir, los sumandos de
(1 + p1 + p21 + . . . + pk11 )(1 + p2 + p22 + . . . + pk22 ) . . . (1 + pr + p2r + . . . + pkr r ).
Claramente hay (k1 + 1)(k2 + 1) . . . (kr + 1) sumandos por lo que
(pk11 pk22 . . . pkr r ) = (k1 + 1)(k2 + 1) . . . (kr + 1)
y
(pk11 pk22 . . . pkr r ) = (1 + p1 + p21 + . . . + pk11 )(1 + p2 + p22 + . . . + pk22 ) . . . (1 + pr + p2r + . . . + pkr r )
29

o equivalentemente
(pk11 pk22 . . . pkr r ) =

pkr +1 1
p1k1 +1 1 p2k2 1+1 1
... r
.
p1 1
p2 1
pr 1

Con estas descripciones, si mcd(m, n) = 1, eso quiere decir que sus factorizaciones son,
como en la demostracion del Teorema 3.9, de la forma m = pk11 pk22 . . . pkr r y n = q1l1 q2l2 . . . qsls ,
con pi = qj para todo i = 1, 2, . . . , r y j, 1, 2, . . . , s. Por tanto, mn factoriza como mn =
pk11 pk22 . . . pkr r q1l1 q2l2 . . . qsls , y se tiene
(mn) = (k1 + 1)(k2 + 1) . . . (kr + 1)(l1 + 1)(l2 + 1) . . . (ls + 1) = (m) (n)
y
pkr +1 1 q1l1 +1 1 q2l2 1+1 1
q ls +1 1
p1k1 +1 1 p2k2 1+1 1
... r
... r
= (m)(n)
p1 1
p2 1
pr 1
q1 1
q2 1
qs 1

(mn) =

En el Ejemplo 3.10 hemos visto que, aplicando la construcci


on
ejemplo 2 genera el 3 y que el 3 genera la funci
on , mientras que el
funci
on y esta generado por la funci
on . Cabe preguntarse, por
2 esta generado a partir de alguna funci
on aritmetica. El siguiente
respuesta:

del Teorema 3.9, el


ejemplo 4 genera la
tanto, si el ejemplo
resultado nos da la

on denida por:
Proposici
on 3.12. Sea : Z1 {1, 0, 1} la funci

1
(n) =
0

(1)r

si n = 1
si existe un primo p tal que p2 |n
si n = p1 p2 . . . pr con p1 , p2 , . . . , pr primos distintos

Entonces
(i) es multiplicativa.
(ii)

d|n

(d) =

1
0

si n = 1
si n > 1.

Demostraci
on: Veamos primero la parte (i). Sean por tanto m, n Z1 con mcd(m, n) =
1. Distinguimos tres casos:
Si m = 1 o n = 1, es evidente que (mn) = (m)(n), ya que (1) = 1.
Si existe un primo p cuyo cuadrado divide a m (resp. n), entonces su cuadrado divide
tambien a mn, luego (mn) = 0, que conicide con (m)(n) ya que (m) (resp. (n)) es
cero.
30

Finalmente si ni m ni n son divisibles por cuadrados de primos, se podr


a escribir
m = p1 . . . pr y q1 . . . qs , con p1 , . . . , pr , q1 , . . . , qs primos distintos. Por tanto, mn =
p1 . . . pr q1 . . . qs y se tendr
a
(mn) = (1)r+s = (1)r (1)s = (m)(n).
a multiplicativa por (i) y
Para la parte (ii), denimos F (n) =
d|n (d), que ser
el Teorema 3.9. Por tanto, basta comprobar que F (1) = 1 (lo que es evidente, ya que
ltimo
F (1) = (1) = 1) y que si p es un n
umero primo y k > 0 entonces F (pk ) = 0. Esto u
k
2
k
j
on (p ) = 0 si j 2,
sigue de que los divisores de p son 1, p, p , . . . , p , y que por denici
por lo que
F (pk ) = (1) + (p) + (p2 ) + . . . + (pk ) = 1 + (1)1 + 0 + . . . + 0 = 0.

Denici
on. La funci
on denida en el teorema anterior se llama funci
on de M
obius.
Teorema 3.13. Sea f una funci
on aritmetica (no necesariamente multiplicativa) y sea F
la funci
on denida por F (n) = d|n f (d). Entonces:
(i) (F
ormula de inversi
on de M
obius) f (n) =
(ii)

n
i=1

F (i) =

n
k=1

n
k

d|n

(d)F ( nd ) =

d|n

( nd )F (d).

f (k).

Demostraci
on: Es claro que, en (i), las dos expresiones para f (n) son iguales, as que basta
demostrarlo para la primera expresi
on. Tendremos entonces:

d|n

n
(d)F ( ) =
d

(d)
d|n

f (c) =
c| n
d

(d)f (c).
d|n,c| n
d

La primera observaci
on ahora es que el conjunto de pares (d, c) tales que d|n y c| nd es igual
al conjunto de pares (d, c) tales que c|n y d| nc . Por tanto, la igualdad anterior prosigue
como
n
(d)F ( ) =
(d)f (c) =
f (c)
(d) .
d
n
n
d|n

c|n,d| c

c|n

Ahora bien, por la Proposici


on 3.12 sabemos que
(d) =
d| n
c

por lo que

c|n

f (c)

d| n
c

1
0

(d) = f (n).
31

si
si

n
c
n
c

=1
> 1,

d| c

Para demostrar (ii), tenemos que


n

F (i) =
i=1

f (d) =
i=1

f (d).
i=1 d|i

d|i

La observaci
on central es que basta demostrar que, para cada k = 1, . . . , n, f (k) aparece
n
a tantas veces como k sea un divisor
k veces en la suma anterior. Y en efecto, k aparecer
de un n
umero i {1, 2, . . . , n}. Como k es divisor de 1 k, 2 k, . . . , nk k, el resultado sigue
inmediatamente.
Ejemplo 3.14. Aplicando el resultado anterior a los casos 3, 4, y 5 del Ejemplo 3.10,
deducimos respectivamente las siguientes formulas para todo n Z1 :
Si f = 1 y F = :
d|n

n
( ) (d) = 1
d

(i) =
i=1

k=1

si f = id y F = :
d|n

n
,
k

n
( )(d) = n
d

(i) =
i=1

k=1

n
k
k

y si f = y F = id:
(n) = n
d|n
n

(d)
=
d
n

i=
i=1

k=1

d|n

n
( )d
d

n
(k)
k

Veamos alguna aplicaci


on de las funciones y en teora de n
umeros.
Ejemplo 3.15. La primera observaci
on es que es equivalente que un n
umero n sea primo
a que (n) = 2 o (n) = 1 + n. Las f
ormulas anteriores nos dan en realidad una f
ormula
para calcular estas funciones para un n
umero n del que no sabemos si factoriza. En efecto,
n
n1
la observaci
on es que en general F (n) = i=1 F (i) i=1 F (i), as que, por ejemplo,
tendremos la f
ormula, para n > 1,
n

(n) =
k=1

n1

k=1

n1
=2+
k
32

n1

k=2

n
n1

.
k
k

Por ejemplo, para n = 7, podemos escribir


(7) = 2 + (3 3) + (2 2) + (1 1) + (1 1) + (1 1) = 2,
lo que concluye que 7 es primo. En realidad, el resultado no es sorprendente, ya que n > 1
es primo si y solo si no es divisible por ning
un k = 2, . . . , n 1, es decir, que nk no es
, es decir (n) = 2.
entero, que es equivalente a nk = n1
k
Veamos otra simple aplicacion del tipo de ideas anterior para estudiar una clase de
n
umeros que llamaron la atenci
on a los griegos clasicos:
Denici
on. Se llama n
umero perfecto a un entero positivo que sea igual a la suma de todos
sus divisores menores que el. Usando la terminologa anterior, la suma de los divisores de n
es (n), por lo que para obtener la suma de los divisores de n menores que n hay que quitar
el propio n. Es decir, n es un n
umero perfecto si y solo si (n) n = n, i.e. (n) = 2n.
El resultado m
as importante sobre los n
umeros perfectos es el siguiente:
Teorema 3.16. Un n
umero par n es un n
umero perfecto si y solo si se puede escribir
k1 k
k
(2 1), siendo k 2 y 2 1 un n
umero primo.
como n = 2
Demostraci
on: Supongamos en primer lugar que n es un n
umero perfecto par. Lo escribimos separando los factores primos impares, es decir, de la forma n = 2k1 m con k 2 y
m impar. La condici
on de que n sea perfecto es equivalente a (n) = 2n, que usando que
es multiplicativa y la f
ormula de la Observaci
on 3.11 quiere decir:
2k m = 2n = (n) = (2k1 m) = (2k1 )(m) = (2k 1)(m).
Como mcd(2k , 2k 1) = 1, se sigue de que existe M Z1 tal que
m = (2k 1)M
(m) = 2k M
En particular, m y M son dos divisores distintos de m, por lo que se tendr
a (m) m+M ,
y de hecho se dara la igualdad si y s
olo si m y M son los u
nicos divisores de m, es decir,
m es primo y M = 1. Ahora observamos que efectivamente se da la igualdad, ya que se
tiene
2k M = (m) m + M = (2k 1)M + M = 2k M.
umero primo y n = 2k1 m = 2k1 (2k 1).
Por tanto, m = 2k 1 es un n
Recprocamente, si n = 2k1 (2k 1), con k 2 y 2k 1 = p primo, entonces, como
es multiplicativa y usando la f
ormula de la Observaci
on 3.11, se tiene
(n) = (2k1 p) = (2k1 )(p) = (2k 1)(p + 1) = (2k 1)2k = 2n.

33

El interes del resultado anterior radica en que no se conocen n


umeros perfectos impares, y de hecho todo indica que no existen (por ejemplo, se sabe que un n
umero perfecto
impar debera tener al menos 300 cifras). Respecto a la existencia de innitos n
umeros
perfectos pares, es tambien un problema abierto. Por el teorema anterior, bastara que deon
mostrar que existen innitos n
umeros primos de la forma 2k 1. Es una simple observaci
k
s
k
que si 2 1 es primo entonces k es primo (ya que, si r|k =, entonces 2 1|2 1).
Denici
on. Se llama n
umero primo de Mersenne a un n
umero primo de la forma Mk :=
k
2 1 con k primo (n
otese que es equivalente a decir que, en base dos, se escribe solo con
unos).
Por tanto, existen innitos n
umeros perfectos pares si y solo si existen innitos
n
umeros primos de Mersenne (lo que se conjetura que es cierto). Damos a continuaci
on
umero
una tabla con los primeros k primos tales que Mk es primo, con el correspondiente n
k1 k
(2 1).
perfecto n = 2
k

Mk = 2k 1

28

31

496

127

8128

13

8191

33550336

17

131071

8589869056

19

524287

137438691328

31

2147483647

2305843008139952128

61

n = 2k1 (2k 1)

2305843009213693951 2658455991569831744654692615953842176

Notese que k = 11 es el primer n


umero primo tal que Mk no es primo (M11 = 2047 =
23 89), y a partir de ah hay muchos k con la misma propiedad. Hasta la fecha, se conocen
solo 43 n
umeros primos de Mersenne (el u
ltimo, obtenido en septiembre de 2008). El
mayor n
umero primo de Mersenne que se conoce (encontrado en agosto de este a
no) es
para k = 43112609, y el n
umero de cifras de Mk es 12978189 (casi trece millones). El
primo M31 ya era conocido por Euler en 1772.
34


4. Ordenes,
races primitivas e ndices
El Ejemplo 3.7 muestra que, dado n Z1 , no siempre (n) es la menor potencia a la
que debemos elevar un n
umero para que nos de 1 modulo n. En este captulo, pretendemos
estudiar en que condiciones (n) es dicho menor exponente. Las deniciones naturales de
partida son las siguientes:
Denici
on. Fijado un entero positivo n, para cada a Z tal que mcd(a, n) = 1 se llama
orden del elemento a m
odulo n al menor entero positivo k tal que ak 1 (mod n). Del
teorema de Euler sigue k (n), y diremos que a es una raz primitiva (de la unidad)
m
odulo n si su orden es k = (n).
Ejemplo 4.1. Estudiemos el caso n = 11. Si tomamos a = 2, es un simple calculo observar
que las potencias 21 , 22 , 23 , 24 , 25 , 26 , 27 , 28 , 29 , 210 son respectivamente congruentes con
2, 4, 8, 5, 10, 9, 7, 3, 6, 1 modulo 11. Por tanto, 2 tiene orden 10 m
odulo 11, y por tanto es
una raz primitiva. Observese que las distintas potencias nos han dado precisamente todos
los restos 1, 2, 3, 4, 5, 6, 7, 8, 9, 10, aunque en orden distinto. Ya veremos m
as adelante que
1 2 3 4 5
esto no es una coincidencia. Si en cambio tomamos a = 3, las potencias 3 , 3 , 3 , 3 , 3 son
respectivamente congruentes con 3, 9, 5, 4, 1 modulo 11, por lo que 3 tiene orden 5 m
odulo
11 y no es una raz primitiva. Tampoco es casualidad que el orden de 3 sea un divisor de
(11) = 10.
No tienen por que existir siempre races primitivas (de hecho, es lo menos frecuente).
Por ejemplo, el Ejemplo 3.7 indica que 35 no tiene races primitivas. M
as a
un, indica que
el problema es que 35 factoriza en dos n
umeros primos entre s, 7 y 5, cuyos valores de
no son primos entre s, ya que son ambos pares. El siguiente resultado indica que esta
situaci
on se repetir
a muchas veces.
Lema 4.2. Si n > 2, (n) es par.
Demostraci
on: Si n contiene alg
un divisor primo impar p, entonces el resultado es evidente,
k
ya que se podr
a escribir n = p m con mcd(m, p) = 1 y k 1. Entonces (n) = (pk )(m),
on 3.4 se tiene (pk ) = pk1 (p 1) y p 1 es par.
y (pk ) es par, ya que por la Observaci
Queda por tanto estudiar el caso en que n no tiene factores impares, es decir, es de la
forma n = 2k . En tal caso, (2k ) = 2k1 , que es tambien par por ser k > 1 (puesto que
n > 2).
Del lema sacamos ya muchos casos en que no hay races primitivas.
Proposici
on 4.3. Si mcd(m, n) > 1 y m, n > 2, entonces mn no tiene races primitivas.
35

Demostraci
on: Sea a Z tal que mcd(a, mn) = 1, es decir, mcd(a, m) = 1 y mcd(a, n) = 1
(ver Proposici
on 1.9). Si aplicamos el Teorema de Euler separadamente a m y n, tendremos
(m)
1 (mod m) y a(n) 1 (mod n). Elevando respectivamente a (n)
y (m)
que a
2
2
(m)(n)

(que son enteros, ya que por el lema (m) y (n) son pares) obtenemos que a 2
es
congruente con 1 m
odulo m y n, luego tambien modulo mn. Por tanto, el orden de a
. Como (mn) = (m)(n) al ser mcd(m, n) = 1, se
modulo mn es a lo sumo (m)(n)
2
tiene que ning
un a tiene orden (mn).
Como consecuencia, los u
nicos n
umeros que pueden tener races primitivas son los de
k
k
la forma p o 2p (con p = 2 en el segundo caso). El caso de las potencias de 2 queda
excluido (salvo para las dos primeras potencias) por el siguiente resultado.
Proposici
on 4.4. Para todo n
umero impar a y para cada entero k 3, se tiene que
2k2
k
a
1 (mod 2 ). Por tanto, 2k no tiene races primitivas.
Demostraci
on: Si demostramos la primera parte, como (2k ) = 2k1 por la Observaci
on
3.4, se sigue que 2k no tiene races primitivas. Basta entonces demostrar la primera parte,
k2
es decir, 2k |a2 1 si k 3. Lo demostraremos por inducci
on sobre k. Si k = 3, entonces
2
tenemos a 1 = (a + 1)(a 1). Como a 1 y a + 1 son dos pares consecutivos, uno de
ellos es necesariamente m
ultiplo de 4, con el producto de ambos es divisible por 8.
Supongamos ahora que lo tenemos demostrado para k y queremos demostrarlo para
k1
k2
k2
1 = (a2
+ 1)(a2
1). El primer factor es par,
k + 1. Entonces escribimos a2
mientras que el segundo, por hip
otesis de induccion, es divisible por 2k , as que el producto
de ambos factores es divisible por 2k+1 , como queramos demostrar.
El resultado nal, que demostraremos algo m
as adelante, cuando dispongamos de las
tecnicas necesarias, es el siguiente:
Teorema 4.5. Un n
umero natural n > 1 posee races primitivas si y solo si es n = 2,
n = 4, n = pk o n = 2pk donde p es un primo impar y k 1.
Por las Proposiciones 4.3 y 4.4, tenemos que los u
nicos casos posibles son los del
enunciado. Por otra parte, es evidente que 3 es una raz primitiva m
odulo 2 y modulo 4.
k
k
Por tanto, basta ver que los n
umeros de la forma p o 2p tienen races primitivas. Eso lo
demostraremos en el Teorema 4.12.
Dado que debemos estudiar la existencia de elementos de orden (n), parece razonable
estudiar primero propiedades de los ordenes de los elementos.
36

Proposici
on 4.6. Sea a un elemento de orden k modulo n. Entonces:
(i) ah 1 (mod n) si y solo si k|h.
(ii) k es un divisor de (n).
(iii) ai aj (mod n) si y solo si i j (mod k).
odulo n.
(iv) Los elementos a, a2 , . . . , ak1 , ak son incongruentes m
(v) Si h > 0, entonces ah tiene orden

k
mcd(h,k)

modulo n.

Demostraci
on: Para la parte (i), sea h = qk + r la divisi
on eucldea de h entre k, por lo
que 0 r < k. Se tendr
a entonces
ah = aqk+r = (ak )r ar 1k ar = ar (mod n).
on de
Por tanto, ah 1 (mod n) si y solo si ar 1 (mod n). Como r < n y, por denici
k
r
orden, k es el menor entero positivo tal que a 1 (mod n), se sigue que a 1 (mod n)
si y s
olo si r 0, es decir, si y solo si r = 0 (ya que r 0). Como esto es equivalente a
decir k|h, queda demostrado (i).
La parte (ii) es una consecuencia de (i), ya que, por el Teorema de Euler, se tiene
1 (mod n).

(n)

Para (iii), podemos suponer i j. Entonces ai (aji 1) 0 (mod n), es decir,


n|ai (aji 1). Como mcd(a, n) = 1, se sigue que n|aji 1, es decir, aji 1 (mod n).
Por (i), k|j i, o lo que es lo mismo i j (mod k).
La parte (iv) es una consecuencia de (iii), ya que, si i, j {1, 2, . . . , k}, la condici
on
j
a a (mod n) es equivalente a i j (mod k), que a su vez es equivalente a i = j, por
ser {1, 2, . . . , k} un sistema completo de restos modulo k (Proposici
on 2.5(i)).
i

Finalmente, para demostrar la parte (v), observamos que (ah )j 1 (mod n) si y solo
si ahj 1 (mod n), que por (i), es equivalente a k|hj. Por el Lema 1.1(iv), esto equivale
k
h
k
h
| mcd(h,k)
j. Como, por la Proposici
on 1.5(ii), mcd(h,k)
y mcd(h,k)
son primos
a mcd(h,k)
entre s, el Lema de Euclides implica que la condici
on anterior es equivalente tambien a
k
h
odulo n
mcd(h,k) |j. En otras palabras, las potencias de a que son congruentes con 1 m
k
k
son precisamente los m
ultiplos de mcd(h,k) , lo que implica que mcd(h,k) es precisamente el
h
orden de a modulo n.
La observaci
on principal a la hora de buscar races primitivas es que estas son las
soluciones de la congruencia x(n) 1 (mod n) que no son soluci
on soluci
on de ninguna
d
congruencia de la forma x 1 (mod n) con d|(n). En los casos que hemos visto
en que n no tiene races primitivas ocurre que existe d < (n) tal que la congruencia
xd 1 (mod n) tiene (n) soluciones modulo n (como en el Ejemplo 3.7, en que la
37

congruencia x12 1 (mod 35) tiene 24 soluciones). Este tipo de anomalas (tener mas
soluciones que el grado de la congruencia) no se dan en el caso en que n es un n
umero
primo (ni para congruencias de grado uno, como indica la Proposici
on 2.9), y de hecho se
tiene el siguiente resultado, que ser
a clave para demostrar el Teorema 4.5:
Teorema 4.7 (Lagrange). Sea p un n
umero primo y f (x) = ad xd +ad1 xd1 +. . .+a1 x+a0
un polinomio con alg
un ai no divisible por p. Entonces existen como mucho d soluciones
distintas m
odulo p de la congruencia f (x) 0 (mod p).
Demostraci
on: Lo hacemos por inducci
on sobre d. Si d = 0, entonces f (x) es una constante
a0 no divisible por p, por lo que f (x) 0 (mod p) no tiene solucion. Supongamos entonces
que tenemos el resultado demostrado para polinomios no nulos de grado menor o igual que
d y demostremoslo para polinomios de grado d + 1. Tomemos por tanto un polinomio
un ai no divisible por p. Distinguimos dos
f (x) = ad+1 xd+1 + ad xd + . . . + a1 x + a0 con alg
casos:
un ai , con i = 0, 1, . . . , d, no es divisible por p. Ademas, las
Si p|ad+1 , entonces alg
soluciones de f (x) 0 (mod p) son las soluciones de ad xd + . . . + a1 x + a0 0 (mod p),
que, por hip
otesis de induccion, son al m
aximo d distintas m
odulo p. Por tanto, en este
caso hay incluso menos de d + 1 soluciones modulo p.
Si en cambio p |ad+1 , demostraremos por reduccion al absurdo que f (x) 0 (mod p)
no puede tener m
as de d + 1 soluciones modulo p. En efecto, supongamos que tuviera d + 2
soluciones distintas, que denotamos por x x1 , x2 . . . , xd+1 , xd+2 (mod p). Consideramos
el polinomio
g(x) = f (x) ad+1 (x x1 ) . . . (x xd+1 ),
que claramente tiene grado como mucho d (ya que el coeciente de xd+1 es cero). Ademas,
no todos sus coecientes son divisibles por p, ya que g(xd+2 ) = ad+1 (xd+2 x1 ) . . . (xd+2
xd+1 ) 0 (mod p) porque es el producto de n
umeros no divisibles por p (Corolario 1.14).
Por tanto, por hip
otesis de inducci
on la congruencia g(x) 0 (mod p) tiene a lo sumo d
soluciones distintas modulo p, mientras que claramente x x1 , x2 . . . , xd+1 (mod p) son
soluciones, lo que nos da la contradicci
on que busc
abamos.

Denici
on. Dado un polinomio f (x) con coecientes enteros, llamaremos races del polinomio m
odulo p a las soluciones de la congruencia f (x) 0 (mod p).
no
Un ejemplo tpico del resultado anterior es el polinomio xp1 1, que por el peque
Teorema de Fermat tiene exactamente p1 races modulo p. Entre este hecho y el teorema
podemos demostrar el siguiente resultado fundamental:
38

Proposici
on 4.8. Para cada d|p1, la congruencia xd 1 0 (mod p) tiene exactamente
d soluciones modulo p.
Demostraci
on: Si escribimos p 1 = dk, entonces podemos escribir
xp1 1 = (xd )k 1 = (xd 1) (xd )k1 + (xd )k2 + . . . + xd + 1 = (xd 1)f (x)
donde f (x) esta denido como el segundo factor, y en particular tiene grado dk d =
p 1 d. Como, por el peque
no teorema de Fermat cada a primo con a es raz modulo
p1
1, entonces tambien es raz de xd 1 o de f (x). Como por el Teorema de
p de x
Lagrange xd 1 tiene como mucho d races modulo p y f (x) tiene como mucho p 1 d,
se sigue que se da la igualdad en ambos casos, es decir, xd 1 tiene exactamente d races
modulo p.
Podemos demostrar ya el resultado principal que nos da los elementos que tenemos
de cada orden, y en particular la existencia de races primitivas modulo p:
Teorema 4.9. Para cada d|p1, existen exactamente (d) enteros incongruentes de orden
d modulo p.
Demostraci
on: Llamamos (d) al n
umero de enteros incongruentes m
odulo p de orden d.
Como el orden de un elemento modulo p es un divisor de (p) = p 1, se tendr
a
(d) = p 1.
d|p1

Por otra parte, como vimos en la parte 5 del Ejemplo 3.10,


(d) = p 1,
d|p1

por lo que bastar


a ver que (d) (d) para cada d|p1. Sea entonces d|p1. Si (d) = 0,
entonces no hay nada que demostrar, as que podemos suponer que existe un elemento a de
odulo
orden d modulo p. Entonces, los elementos a, a2 , . . . , ad son todos incongruentes m
d
p por la Proposici
on 4.6(iv), luego son las d soluciones de x 1 0 (mod p) de la
proposici
on anterior. Por tanto, los elementos de orden d son de la forma ai , y por la
Proposici
on 4.6(v), un elemento ai tiene orden d si y solo si mcd(i, d) = 1 , es decir, hay
exactamente (d) elementos incongruentes de orden d, lo que demuestra el resultado.
Corolario 4.10. Cualquier n
umero primo tiene races primitivas.
Demostraci
on: Es consecuencia del teorema, tomando d = p 1. De hecho, podemos
precisar que hay exactamente (p 1) elementos incongruentes entre s de orden p 1
modulo p, es decir, races primitivas modulo p.
39

Pasar de races primitivas modulo un n


umero primo a una potencia suya se basa en
la siguiente observaci
on:
Lema 4.11. Sea r una raz primitiva m
odulo un n
umero primo p. Entonces, para cualquier
k
k 2, el orden de r modulo p es de la forma (p 1)pj , con j k 1. En particular, r
k2
es una raz primitiva m
odulo pk si y solo si pk no divide a rp (p1) 1.
Demostraci
on: Sea m el orden de r modulo pk . Se tendr
a, en particular, am 1 (mod pk ),
a p 1|m, ya que p 1 es
y en particular am 1 (mod p). Por el Lema 4.6(i), se tendr
el orden de r modulo p (por ser r raz primitiva). Adem
as, por el Lema 4.6(ii) aplicado
k
k
k1
a m|(p ) = p
(p 1). De estas dos condiciones sobre m se
ahora a n = p , se tendr
j
deduce que es de la forma m = (p 1)p , con j k 1.
Para demostrar la segunda parte, es claro ahora que r no es una raz primitiva m
odulo
j
olo si tiene orden m = (p1)p , con j k 2, es decir si y solo si su orden m divide
p si y s
k2
on es equivalente a rp (p1) 1 (mod pk ),
a (p 1)pk2 . Por el Lema 4.6(i), esta condici
k2
es decir, a pk |rp (p1) 1.
k

Este resultado nos permite nalmente terminar de demostrar el Teorema 4.5:


Teorema 4.12. Si p es un n
umero primo impar, pk y 2pk tienen races primitivas para
cada k 1.
Demostraci
on: Ya hemos visto en el Corolario 4.10 que existen races primitivas modulo
p. Demostremos en primer lugar una raz primitiva m
odulo p2 . Para ello, jaremos una
raz primitiva r modulo p, y veamos que, si r no es una raz primitiva m
odulo p2 , entonces
r + p lo es (en realidad valdra cualquier r + ip con mcd(i, p) = 1).
En efecto, si r no es una raz primitiva m
odulo p2 , por el Lema 4.11 sabemos que p2
divide a rp1 1. Por tanto, m
odulo p2 tendremos
(r + p)p1 1 rp1 + (p 1)rp2 p 1 (p 1)rp2 p (mod p2 ).
Como (p 1)rp2 no es divisible por p, entonces (p 1)rp2 p no es divisible por p2 , luego
(r + p)p1 1 tampoco lo es, lo que implica por Lema 4.11 que r + p es una raz primitiva
modulo p2 .
Construida pues una raz primitiva r modulo p2 , veamos que, por ser p impar, r es
tambien una raz primitiva m
odulo pk para cualquier k 2. Por el Lema 4.11, tenemos
k2
que demostrar que, si p2 no divide a rp1 1, entonces pk no divide a rp (p1) 1. Lo
demostraremos por inducci
on sobre k, siendo el resultado trivial para k = 2. Supongamos
ahora cierto el resultado para k y vamos a demostrarlo para k + 1. Por el Teorema de
40

k2

Euler para pk1 , sabemos que rp (p1) 1 es divisible por pk1 , y nuestra hip
otesis de
k
inducci
on indica que no es divisible por p . En otras palabras, podemos escribir
k2

rp

(p1)

= 1 + apk1

con a no divisible por p. Elevando a p, tendremos


k1

rp

(p1)

= (1 + apk1 )p = 1 + apk +

p 2 2k2
a p
+ ...
2

Como p > 2 (es aqu donde usamos que p sea impar), p2 es divisible por p, luego el sumando
p 2 2k2
es divisible por p2k1 y 2k 1 k + 1 por ser k 2. El resto de sumandos en
2 a p
los puntos suspensivos son divisibles por pi(k1) con i 3, y como i(k 1) 3k 3 k + 1
son tambien divisible por pk+1 . Se concluye as que
k1

rp

(p1)

1 + apk (mod pk+1 )


k1

y como apk no es divisible por pk+1 , entonces rp

(p1)

1 tampoco lo es.

Finalmente, dada una raz primitiva m


odulo pk , si es par le a
nadimos pk y seguir
a
k
siendo una raz primitiva m
odulo p , pero ahora impar. Por tanto, existe r impar que es una
k
odulo 2pk . En efecto, en
raz primitiva m
odulo p . Veamos que es tambien raz primitiva m
primer lugar es evidente que mcd(r, 2pk ) = 1, ya que mcd(r, 2) = 1 y mcd(r, pk ) = 1. Por
k
otra parte (2pk ) = (2 1)pk1 (p 1) = pk1 (p 1) = (pk ), luego r(2p ) 1 (mod 2pk )
y no existe a < (2pk ) tal que ra 1 (mod 2pk ), por lo que r tiene orden (2pk ) modulo
pk .

Ejemplo 4.13. El resultado anterior es constructivo. Por ejemplo, vamos a buscar una
raz primitiva m
odulo 242 = 2 112 . Ya sabemos, por el Ejemplo 4.1, que 2 es una raz
primitiva m
odulo 11. Nos preguntamos si ser
a tambien una raz primitiva m
odulo 112 ,
para lo que usaremos el Lema 4.11. Como 210 1 = 1023 = 3 11 31 no es divisible por
odulo 112 (n
otese que, si hubieramos
112 , se sigue que 2 es, en efecto, una raz primitiva m
obtenido que no lo era, la demostraci
on del Teorema 4.12(ii) nos dira autom
aticamente
que 2 + 11 es una raz primitiva). Finalmente, por la demostraci
on de la parte (iv) del
2
odulo 2 112 .
Teorema 4.12, tenemos que 2 + 11 = 123 es una raz primitiva m
Ahora que sabemos que n
umeros tienen raz primitiva, veamos en que podemos usarlo. Observese en primer lugar que, si r es una raz primitiva m
odulo n, entonces por la
2
(n)
son todos no congruentes dos a dos, luego,
Proposici
on 4.6(iv) los elementos r, r , . . . , r
por el Lema 3.1, son congruentes modulo n, en alg
un orden, a los n
umeros 1, 2, . . . , n que
41

son primos con n. Por tanto, cualquier a Z con mcd(a, n) = 1 es congruente a un u


nico
on de logaritmo, damos la siguiente:
ri con i {1, 2, . . . , (n)}. Recordando la denici
Denici
on. Dado un n
umero n con raz primitiva r, se llama ndice de a relativo a r al
menor entero positivo k tal que a rk (mod n) y escribiremos k = indr (a).
Ejemplo 4.14. Si n = 11, sabemos por el Ejemplo 4.1 que r = 2 es una raz primitiva
modulo 11. Adem
as, 1, 2, 3, 4, 5, 6, 7, 8, 9, 10 son, respectivamente, congruentes modulo 11
10 1 8 2 4 9 7 3 6 5
con 2 , 2 , 2 , 2 , 2 , 2 , 2 , 2 , 2 , 2 , por lo que sus ndices relativos a 2 son, respectivamente, 10, 1, 8, 2, 4, 9, 7, 3, 6, 5
El ndice funciona, en efecto, como un logaritmo en base r, y de hecho se tienen las
siguientes propiedades:
Proposici
on 4.15. Si r es una raz primitiva m
odulo n, se verica:
(i) rk a (mod n) si y solo si k indr (a) (mod (n)).
(ii) indr (ab) indr (a) + indr (b) (mod (n)).
(iii) indr (ak ) kindr (a) (mod (n)).
(iv) indr (1) 0 (mod (n)) (y de hecho indr (1) = (n)).
(v) indr (r) = 1.
Demostraci
on: Veamos primero (i). Como por denici
on rindr (a) a (mod n), entonces
on 4.6(iii) se sigue k indr (a) (mod (n))
rk rindr (a) (mod n), y por la Proposici
(recuerdese que el hecho de que r sea raz primitiva m
odulo n se traduce en que el orden
de r modulo n es (n)).
El resto de los apartados se demuestra inmediatamente a partir de (i). Haremos s
olo,
a modo de ejemplo, (ii). Como se tiene
rindr (a)+indr (b) = rindr (a) rindr (b) ab (mod n)
la parte (i) implica indr (a) + indr (b) indr (ab) (mod (n)).
La teora de ndices se usa para resolver cierto tipo de congruencias no lineales, pasando
de congruencias modulo n a congruencias modulo (n). Antes de ver el resultado preciso,
hagamos un ejemplo.
Ejemplo 4.16. Consideremos la congruencia x6 3(mod 11). Evidentemente, cualquier
solucion verica mcd(x, 11) = 11. Por tanto, como 2 es una raz primitiva m
odulo 11
y
as, sabemos que
ver Ejemplo 4.1), se podr
a se podr
a escribir x 2 (mod 11). Adem
8
6y
3 2 (mod 11), por lo que nuestra congruencia es equivalente a 2 28 (mod 11).
42

Por la Proposici
on 4.6(iii), esta congruencia es equivalente a 6y 8 (mod 10). Aplicando
ahora la Proposici
on 2.9, sabemos que esta congruencia tiene dos soluciones, precisamente
y 3, 8 (mod 10). De nuevo por la Proposici
on 4.6(iii), estas soluciones son equivalentes
y
3 8
a 2 2 , 2 (mod 11), es decir, x 8, 3 (mod 11).
Teorema 4.17. Sea n un entero con una raz primitiva r, y sean k 2 y d = mcd(k, (n)).
un b Z1 , entonces
Si a Z verica mcd(a, n) = 1, y por tanto a rb (mod n) para alg
son equivalentes:
(i) La congruencia xk a (mod n) tiene solucion.
(ii) d|b.
(iii) a

(n)
d

1 (mod n).

Ademas, en tal caso, la congruencia tiene exactamente d soluciones modulo n


Demostraci
on: Evidentemente, las soluciones de xk a (mod n) verican mcd(x, n) = 1,
por lo que se podr
a escribir x ry (mod n) para alg
un y Z. En otras palabras, la
congruencia tendr
a solucion si y s
olo si tiene solucion la congruencia
rky rb (mod n)
que, por la Proposici
on 4.6(iii), es equivalente a
ky b (mod (n)).
Por la Proposici
on 2.9, tal congruencia tiene soluci
on si y s
olo si d|b, y ademas en tal caso
el n
umero de soluciones modulo (n) es d. Esto da la equivalencia de (i) y (ii), y adem
as
tomando potencias de r nos dice que el n
umero de soluciones de nuestra congruencia
modulo n es d.
Finalmente, d|b es equivalente a (n)|b (n)
on 4.6(i), es equivad , que por la Proposici
(n)

lente a rb d 1 (mod n), es decir, a


son equivalentes.

(n)
d

1 (mod n), lo que demuestra que (ii) y (iii)

El resultado anterior nos permite generalizar el Teorema 2.19:


Teorema 4.18. Sea p un n
umero primo impar, y sea m 1. Entonces la congruencia
2m
olo si p 1 (mod 2m+1 ).
x 1 (mod p) tiene solucion si y s
Demostraci
on: Si escribimos p 1 = 2n a con a impar, entonces es evidente que d =
max{0,nm}
a. Por la parte (iii) del teorema
mcd(2m , p 1) = 2min{n,m} y por tanto p1
d =2
(que podemos usar ya que p tiene races primitivas), la congruencia tendr
a solucion si y
p1
p1
solo si (1) d 1 (mod p), es decir, si y solo si d es par, si y solo si n > m, si y s
olo
m+1
m+1
|p 1, si y solo si p 1 (mod 2
).
si 2
Podemos por tanto generalizar el Corolario 2.20:
43

Teorema 4.19. Para cada m 1, existen innitos n


umero primos de la forma 2m k + 1.
nicos primos de la forma 2m k +1, consideramos
Demostraci
on: Si p1 , p2 , . . . , pr fueran los u
m1
el n
umero n = (2p1 p2 . . . pr )2
+ 1. Claramente, n es impar, luego sus posibles divisores
primos son impares. Si p es un divisor primo impar de n, entonces 2p1 p2 . . . pr es una
m1
solucion de la congruencia x2
1 (mod p). Por el Teorema 4.18, p 1 (mod 2m ),
es decir, p es de la forma 2m k + 1. Por tanto p tiene que ser alguno de los p1 , p2 , . . . , pr , lo
que es absurdo.
Ejercicio 4.20. Demostrar que, para cada m 2, existen innitos n
umero primos de la
m
m1
+ 1.
forma 2 k + 2

44

5. Congruencias cuadr
aticas
En el Teorema 4.17 hemos visto que el Teorema 4.12 parece funcionar particularmente
bien para ciertas congruencias de grado 2k . En este captulo vamos a centrarnos en las
congruencias de grado dos. Observese en primer lugar que cualquier congruencia m
odulo
un entero n se puede descomponer en congruencias modulo las potencias de primos que
dividen a n. Podemos usar por tanto que, con la excepci
on de las potencias de dos, las
potencias de un primo tienen races primitivas. De todas formas, vamos a iniciar nuestro
estudio con congruencias cuadr
aticas modulo un n
umero primo. Podemos suponer que el
2
aticas modulo dos
primo es impar, ya que x x (mod 2), luego las congruencias cuadr
son equivalentes a congruencias lineales.
El primer resultado nos da la cl
asica resolucion de una ecuaci
on de grado dos, pero
ahora en el lenguaje de congruencias.
Proposici
on 5.1. Sea p un n
umero primo impar y sean a, b, c Z tales que mcd(a, p) = 1.
2
Entonces la congruencia ax + bx + c 0 (mod p) tiene solucion si y solo si la congruencia
as, las soluciones de ax2 + bx + c 0 (mod p)
x2 b2 4ac (mod p) tiene solucion. Adem
son exactamente las soluciones de 2ax b + d (mod p) y 2ax b d (mod p), donde
d es una solucion de x2 b2 4ac (mod p).
Demostraci
on: Como mcd(4a, p) = 1, la congruencia ax2 + bx + c 0 (mod p) es equivalente a 4a2 x2 + 4abx + 4ac 0 (mod p), es decir,
(2ax + b)2 b2 4ac (mod p).
Por tanto, si nuestra congruencia original tiene soluci
on, entonces b2 4ac es un cuadrado
modulo p. Recprocamente, si d2 b2 4ac (mod p), entonces la congruencia anterior
es equivalente a p|(2ax + b)2 d2 , es decir p|(2ax + b + d)(2ax + b d), que es a su vez
equivalente a 2ax b + d (mod p) o 2ax b d (mod p), y ambas tienen soluci
on
u
nica porque mcd(2a, p) = 1 (ver Proposici
on 2.9).
El resultado anterior indica que para saber resolver ecuaciones cuadr
aticas modulo un
n
umero primo basta saber decidir cu
ando un n
umero es un cuadrado m
odulo dicho n
umero
primo. Por supuesto, si el n
umero es divisible por el n
umero primo, es congruente con el
cuadrado de cero, y es la u
nica soluci
on. En caso contrario, podemos usar el Teorema 4.17
para dar una respuesta:
Teorema 5.2 (Criterio de Euler). Sea p un n
umero primo impar y sea a Z tal que
p1
2
olo si a 2
mcd(a, p) = 1. Entonces la congruencia x a (mod p) tiene solucion si y s
p1
1 (mod p). Ademas, si la congruencia no tiene soluci
on se verica a 2 1 (mod p)
45

Demostraci
on: Como (p) = p 1 es par, se tiene que mcd(2, (p)) = 2, luego por el
p1
olo si a 2 1 (mod p).
Teorema 4.17 la congruencia x2 a (mod p) tiene solucion si y s
Por otra parte, por el Teorema de Euler se tiene ap1 1 (mod p), es decir,
p|ap1 1 = (a
lo que implica que a

p1
2

p1
2

)2 1 = (a

1 (mod p) o a

p1
2

p1
2

+ 1)(a

p1
2

1)

1 (mod p).

Denici
on. Si mcd(a, p) = 1, se dice que a es un resto cuadr
atico m
odulo p si la congru2
umero
encia x a (mod p) tiene solucion. Se llama smbolo de Legendre al n
a
p

1 si es un resto cuadr
atico modulo p
1 si a no es un resto cuadr
atico modulo p

Para p = 2, se tiene obviamente a2 = 1 para cualquier a impar. Si p es un n


umero primo
impar, por el Teorema 5.2, se tiene
a

p1
2

a
p

(mod p)

Proposici
on 5.3. El smbolo de Legendre verica las siguientes propiedades:
(i) Si a b (mod p) entonces
(ii)

ab
p

a
p

b
p

(iii)

a2
p

= 1, y en particular

(iv)

1
p

= (1)

a
p

b
p

p1
2

1
p

=1

si p es impar.

Demostraci
on: La parte (i) es trivial. La parte (ii) (si p es impar, ya que el caso p = 2 es
trivial) se sigue de que
ab
p

(ab)

p1
2

=a

p1
2

p1
2

a
p

b
p

(mod p)

y ap pb , ya que ambos valen 1 o 1. La parte (iii) es


(ii) cuando a = b, y la parte (iv) es el criterio de Euler (n
otese que es tambien equivalente
al Teorema 2.19).

lo que implica la igualdad de

ab
p

Observaci
on 5.4. La parte (ii) es mucho m
as potente de lo que aparenta. Por ejemplo,
23
podra parecer que no sirve para calcular 31 , pero basta por ejemplo escribir
23
31

8
31

(1)23
31
46

1
31

2
31

2
2
Por (iv), tenemos 1
= 31
= 1, mientras que por (iii) tenemos 31
, por lo que
31
23
2
nalmente tenemos 31 = 31 , que es mas facil de calcular. En efecto, basta utilizar
el criterio de Euler, y observar que

2
por lo que

2
31

311
2

= 215 = (25 )3 = 323 13 = 1 (mod 31)

= 1 y por tanto

23
31

= 1.

Teorema 5.5. Sea p un n


umero primo impar y sea a Z tal que mcd(a, p) = 1. Para
p1
cada k = 1, 2, . . . , 2 , sea ka = qk p + rk la divisi
on de ka entre p. Separamos los restos
r1 , r2 , . . . , r p1 de la forma
2

{r1 , . . . , r p1 } = {r1 , . . . , rm } {r1 , . . . , rn }


2

donde r1 , . . . , rm son los restos menores que


Entonces:

p
2

y r1 , . . . , rn son los restos mayores que

p
2.

(i) {1, 2 . . . , p1
2 } = {r1 , . . . , rm } {p r1 , . . . , p rn }
(ii) (Lema de Gauss)

a
p

= (1)n , donde n es como antes, es decir, el n


umero de ele-

p
mentos de a, 2a, 3a, . . . , p1
2 a cuyo resto al dividir por p es mayor que 2 .

Demostraci
on: Usaremos repetidamente la Proposicion 2.5. Como el conjunto

p1 p3
p1
,
, . . . , 1, 0, 1, . . . ,
2
2
2

esta formado por p n


umeros consecutivos, es un sistema completo de restos modulo p. Por
tanto, tambien los n
umeros

p3
p1
p1
a,
a, . . . , a, 0, a, . . . ,
a
2
2
2

forma un sistema completo de restos modulo p. Como los n


umeros a, . . . , p1
2 a son congruentes modulo p (en alg
un orden), a r1 , . . . , r p1 , es decir, a r1 , . . . , rm , r1 , . . . , rn , se
2
tendr
a que los n
umeros
r1 , . . . , rm , r1 , . . . , rn , 0, r1 , . . . , rm , r1 , . . . , rn
forman un sistema completo de restos modulo p. Esto implica en particular que los n
umeros
r1 , . . . , rm , r1 , . . . , rn son incongruentes m
odulo p cuando se toman dos a dos, y lo
mismo es cierto con los n
umeros r1 , . . . , rm , pr1 , . . . , prn , que seran por tanto distintos.
p+1
Como hay m + n = 2 de estos n
umeros, y todos ellos estan en el conjunto {1, 2 . . . , p1
2 },
se sigue inmediatamente la parte (i).
47

Multiplicando entonces todos estos n


umeros, se obtiene:
p1
! = r1 . . . rm (p r1 ) . . . (p rn ) (1)n r1 . . . rm r1 . . . rn (mod p).
2
y, como por otra parte,
r1 . . . rm r1 . . . rn a(2a)(3a) . . . (
juntando ambas congruencias, se sigue a

p1
2

p1
a) =
2

p1
p1
!a 2 (mod p),
2

(1)n (mod p), lo que prueba (ii).

Teorema 5.6. Si p es un primo impar, entonces


2
p

1 si p 1 (mod 8) o p 7 (mod 8)
1 si p 3 (mod 8) o p 5 (mod 8)

Demostraci
on: Usamos el lema de Gauss, y tenemos que ver cuantos de los p1
umeros
2 n
p
2, 4, . . . , p 1 son mayores que 2 . Lo haremos analizando separadamente todos los posibles
restos de p modulo 8:
Si p = 8k +1, entonces el conjunto 2, 4, . . . , p1 = 8k tiene tenemos 4k elementos, de
los que los menores de p2 = 4k+ 12 son 2, 4, . . . , 4k, es decir, 2k. Por tanto, n = 4k2k = 2k,
que es par, luego

2
p

= 1.

Si p = 8k + 3, entonces el conjunto 2, 4, . . . , p 1 = 8k + 2 tiene tenemos 4k + 1


elementos, de los que los menores de p2 = 4k + 32 son 2, 4, . . . , 4k, es decir, 2k. Por tanto,
n = (4k + 1) 2k = 2k + 1, que es impar, luego

2
p

= 1.

Si p = 8k + 5, entonces el conjunto 2, 4, . . . , p 1 = 8k + 4 tiene tenemos 4k + 2


elementos, de los que los menores de p2 = 4k + 52 son 2, 4, . . . , 4k + 2, es decir, 2k + 1. Por
tanto, n = (4k + 2) (2k + 1) = 2k + 1, que es impar, luego

2
p

= 1.

Si p = 8k + 7, entonces el conjunto 2, 4, . . . , p 1 = 8k + 6 tiene tenemos 4k + 3


elementos, de los que los menores de p2 = 4k + 72 son 2, 4, . . . , 4k + 2, es decir, 2k + 1. Por
tanto, n = (4k + 3) (2k + 1) = 2k + 2, que es par, luego

2
p

= 1.

Corolario 5.7. Existen innitos n


umeros primos de la forma 8k + 7.
Demostraci
on: Si p1 , p2 , . . . , pr fueran todos los n
umeros primos de la forma 8k + 7,
2
consideramos el n
umero a = 8(p1 p2 . . . pr ) 1. Sea p un divisor primo de a (necesariamente
p sera impar). Entonces, (4p1 p2 . . . pr )2 2 (mod p), por lo que, por el Teorema 5.6, se
tiene p 1 (mod 8) o p 7 (mod 8). Como a 7 (mod 8), no todos los divisores de a
48

pueden ser de la forma 8k + 1. Por tanto, a es divisible por alg


un primo de la forma 8k + 7,
es decir, por alg
un pi , lo que es absurdo.
Teorema 5.8. Si p es un primo impar, entonces
2
p

1 si p 1 (mod 8) o p 3 (mod 8)

1 si p 5 (mod 8) o p 7 (mod 8)

Demostraci
on: Basta combinar el Teorema 5.6 con las partes (iii) y (iv) de la Proposici
on
5.3. Lo hacemos seg
un el resto de p modulo 8:
Si p 1 (mod 8), entonces por el Teorema 5.6,

2
p

= 1, mientras que por otra


1
p

parte p 1 (mod 4) luego por la Proposici


on 5.3(iv) se tiene
Proposici
on 5.3(iv) se sigue

2
p

= 1.

Si p 3 (mod 8), entonces por el Teorema 5.6,

= 1, mientras que por otra

2
p

parte p 3 (mod 4) luego por la Proposici


on 5.3(iv) se tiene
la Proposici
on 5.3(iv) se sigue

2
p

1
p

= 1. Por tanto, de la

= 1.

Si p 7 (mod 8), entonces por el Teorema 5.6,

2
p

= 1, mientras que por otra

parte p 3 (mod 4) luego por la Proposici


on 5.3(iv) se tiene
la Proposici
on 5.3(iv) se sigue

= 1. Por tanto, de

= 1, mientras que por otra

2
p

parte p 1 (mod 4) luego por la Proposici


on 5.3(iv) se tiene
2
p

1
p

= 1.

Si p 5 (mod 8), entonces por el Teorema 5.6,

Proposici
on 5.3(iv) se sigue

= 1. Por tanto, de la

2
p

1
p

= 1. Por tanto, de

= 1.

Corolario 5.9. Existen innitos n


umeros primos de la forma 8k + 3.
umeros primos
Demostraci
on: Como siempre, supogamos que p1 , p2 , . . . , pr son todos los n
2
de la forma 8k + 3. Consideramos el n
umero a = (p1 p2 . . . pr ) + 2. Entonces, si p es un
divisor primo de a (necesariamente impar), se tendra que (p1 p2 . . . pr )2 2 (mod p), y
por el Teorema 5.8 se tendra que p es congruente con 1 o 3 modulo 8. Como a es de la
forma 8k + 3 (ya que el cuadrado de cada pi es de la forma 8k + 1), no todos los divisores
de a son de la forma 8k + 1. Por tanto, a es divisible por alg
un n
umero primo de la forma
8k + 3, es decir, por alg
un pi ; lo que es imposible.
Damos a continuaci
on un modo pr
actico de calcular los smbolos de Legendre:
49

Lema 5.10. Si p es un primo impar, a Z es impar y mcd(a, p) = 1, entonces


a
p

= (1)

p1
2
k=1

[ ka
p ].

Demostraci
on: Usando las notaciones del Teorema 5.5, tenemos que demostrar, por la
p1

ka
2
tienen la misma paridad. Como a y p son
parte (ii) de dicho teorema, que n y k=1
p
impares, y tomando congruencias m
odulo 2, tendremos:
p1
2

k=1

ka
p
p

p1
2

k=1

ka
=p
p

p1
2

p1
2

qk =
k=1

p1
2

ka(r1 +. . .+rm )(r1 +. . .+rn )

(kark ) =
k=1

k=1

p1
2

k (r1 + . . . + rm ) (r1 + . . . + rn ) (mod 2).


k=1

Por el Teorema 5.5(i), {r1 , . . . , rm , p r1 , . . . , p rn } = {1, 2, . . . , p1


2 }, y por tanto
p1
2

(r1 + . . . + rm ) + ((p r1 ) + . . . + (p rn )) =

k.
k=1

Sustituyendo en la congruencia anterior, obtenemos


p1
2

k=1

ka
((p r1 ) + . . . + (p rn )) (r1 + . . . + rn ) = pn 2(r1 + . . . + rn ) n (mod 2)
p

que es lo que queramos demostrar.


El resultado anterior nos permite demostrar el siguiente resultado fundamental, que
servir
a para calcular cualquier smbolo de Legendre:
Teorema 5.11 (Ley de reciprocidad cuadr
atica). Si p, q son primos impares distintos,
entonces
p1 q1
q
p
= (1) 2 2 .
q
p
Demostraci
on: Consideramos el conjunto de pares de la forma (k, l), con k {1, 2, . . . , p1
2 }
q1
p1 q1
y l {1, 2, . . . , 2 }. Evidentemente, hay 2 2 de dichos pares. Dividimos ahora el
mencionado conjunto en dos subconjuntos:
En primer lugar, consideramos los pares (k, l) tales que kq lp. Para cada l
{1, 2, . . . , q1
2 } los posibles valores de k son 1, 2, . . . ,
de esta forma es

q1
2

l=1

lp
q

.
50

lp
q

. Por tanto, el n
umero de pares

Consideramos por otra parte los pares (k, l) tales que kq lp. Para cada k
kq
umero de pares
{1, 2, . . . , p1
2 } los posibles valores de l son 1, 2, . . . , p . Por tanto, el n
de esta forma es

p1
2

k=1

kq
p

Observemos que no hay pares que esten en ambos subconjuntos, ya que por ejemplo
p no divide a ning
un kq. Por tanto, el n
umero total de pares es igual, por una parte a
p1 q1
2
2 ,

y por otra parte a


el Lema 5.10, tenemos
(1)

p1 q1
2
2

= (1)

p1
2
k=1

p1
2

k=1

[ ka
p ]+

kq
p

q1
2
l=1

q1
2

l=1

lp
q

[ lpq ] = (1)

. Tomando potencias de 1 y usando

p1
2
k=1

q1
2
l=1

[ ka
p ] (1)

[ lpq ] =

q
.
p

p
q

Observaci
on 5.12.
La ley de reciprocidad cuadr
atica es el resultado del que mas
demostraciones se conocen. En http://www.rzuser.uni-heidelberg.de/hb3/fchrono.html
hay una lista de ellas, y hasta el a
no 2007 se llevan contabilizadas 224 distintas.
Corolario 5.13. Sea p > 3 un n
umero primo. Entonces:
(i)

(ii)

3
p

3
p

1 si p 1 (mod 12) o p 11 (mod 12)


1 si p 5 (mod 12) o p 7 (mod 12)
1 si p 1 (mod 3)
1 si p 2 (mod 3)

Demostraci
on: Por la ley de reciprocidad cuadr
atica, tenemos
otra parte, por la Proposici
on 5.3(iv), tenemos
1
p

3
p

1
p

= (1)

3
p
p1
2

= (1)

p1
2

. Por tanto,

p
3

. Por
3
p

p
3

, que claramente viene dado por la f


ormula de (ii). La parte (i) se
demuestra como el Corolario 5.8 a partir de (i).
Corolario 5.14. Existen innitos n
umeros primos de la forma 12k + 11.
Demostraci
on: Supongamos en primer lugar que p1 , . . . , pr sean todos los n
umeros primos
2
de la forma 12k+11. Consideramos a = 12(p1 . . . pr ) 1. Entonces, si p es un divisor primo
de a (necesariamente p > 3), se tiene que 6p1 . . . pr es una soluci
on de x2 3 (mod p).
El Corolario 5.13(i) implica entonces que p es de la forma 12k + 1 o 12k + 11. Como a es
de la forma 12k + 11, necesariamente tendra alg
un divisor primo de la forma 12k + 11, es
decir, alg
un pi , lo que es absurdo.
51

Ejercicio 5.15. Demostrar que existen innitos n


umeros primos de la forma 3k + 1. M
as
a
un, demostrar que existen innitos n
umeros primos de la forma 12k + 7.
Ejemplo 5.16. Veamos en un ejemplo pr
actico como la Ley de Reciprocidad Cuadr
atica
permite calcular cualquier smbolo de Legendre. Supongamos que queremos calcular 17
41 .
17
41
Por la Ley de Reciprocidad Cuadr
atica, podremos escribir 41 = 17 y, como se tiene
7
41 7 (mod 17), tambien tendremos 41
17 = 17 . Por tanto, hemos reducido calcular un
7
smbolo 17
umeros mas peque
nos. Es claro que podemos
41 a calcular 17 , que involucra n
7
17
3
seguir el proceso, escribiendo ahora 17 = 7 = 7 , que es igual a 1 por el Corolario
5.13. Por tanto, 17
41 = 1.
Aprovechamos para hacer el mismo calculo con un truco distinto. Podemos escribir
17
41

24
41

2 22 3
41

2
41

22
41

3
41

= 1 1 (1) = 1

donde los tres u


ltimos smbolos han sido calculados respectivamente con el Teorema 5.6,
la Proposici
on 5.3(iii) y el Corolario 5.13(i).
Veamos, para terminar el captulo, que el smbolo de Legendre tambien mide cuando
un n
umero es un resto cuadr
atico modulo la potencia de un primo:
Teorema 5.17. Si p es un primo impar, k 1, y mcd(a, p) = 1, entonces la congruencia
x2 a (mod pk ) tiene solucion si y s
umero de
olo si ap = 1. Ademas, en tal caso, el n
soluciones modulo pk es dos.
Demostraci
on: Obviamente, si x2 a (mod pk ) tiene solucion, tambien la tiene x2
a (mod p), por lo que ap = 1. Hay que demostrar entonces el recproco, que si ap = 1
on sobre k, siendo trivial
entonces x2 a (mod pk ) tiene solucion. Lo haremos por inducci
el caso k = 1.
Supongamos entonces que x2 a (mod pk ) tiene solucion x0 y encontremos una
soluci
on de x2 a (mod pk+1 ). Como las soluciones de esta segunda congruencia son
soluciones de la primera, en realidad buscamos enteros de la forma x0 + ypk que sean
solucion de la segunda congruencia.
x20

Que x0 sea solucion de la primera congruencia quiere decir que se puede escribir
= a + bpk , para alg
un b Z. Por tanto, tendremos
(x0 + ypk )2 = x20 + 2x0 ypk + y 2 p2k = a + (2x0 y + b)pk + p2k

de donde se deduce que (x0 + ypk )2 a (mod pk+1 ) si y solo si p|2x0 y + b. Como
mcd(2, p) = 1, la congruencia 2x0 y +b 0 (mod p) tiene solucion, y tomando una soluci
on
k
2
k+1
y se tiene que x0 + yp es una solucion de x a (mod p
).
52

El hecho de que haya exactamente dos soluciones modulo pk es consecuencia del Teorema 4.17, ya que pk tiene races primitivas. Puede hacerse tambien directamente, ya que,
si x0 es una solucion, entonces cualquier otra soluci
on x verica x2 x20 (mod pk ), es decir,
aneamente p|x+x0 y p|xx0 , ya que
pk |x2 x20 = (x+x0 )(xx0 ). No puede ocurrir simult
entonces p|(x + x0 ) (x x0 ) = 2x0 y, como p es un primo impar, p|x0 , lo que contradice
que x20 1 (mod pk ). Por tanto mcd(pk , x + x0 ) = 1 o bien mcd(pk , x x0 ) = 1, por lo
que pk |x x0 o bien pk |x + x0 . Por tanto, x x0 (mod pk ) o bien x x0 (mod pk ), y
estas son las u
nicas soluciones.

Ejemplo 5.18. Veamos como se puede seguir la demostracion anterior para resolver
3
congruencias cuadr
aticas. Por ejemplo, como 13
= 1 (por el Corolario 5.13(i)), entonces
2
2
a dos soluciones modulo 132 . En primer lugar, debemos
la ecuacion x 3 (mod 13 ) tendr
resolver la ecuacion x2 3 (mod 13), y se ve a ojo que x 4 (mod 13) son las soluciones.
Trabajamos por elemplo con la soluci
on x0 = 4, y tenemos que buscar ahora los valores
de y para los que 4 + 13y es solucion de x2 3 (mod 132 ). Operando (mejor as que
intentar aprenderse de memoria la f
ormula) queremos 16 + 104y + 132 y 2 3 (mod 132 ), es
decir, 13(1 + 8y) 0 (mod 132 ), que es equivalente a 1 + 8y 0 (mod 13). Congruencias
lineales de este tipo ya aprendimos a resolverlas en el Captulo 2, y se comprueba f
acilmente
que y 8 (mod 13) es la solucion. Por tanto, x 4 + 13 8 = 108 (mod 132 ) es una
solucion. Para calcular la segunda soluci
on no hace falta empezar con x0 = 4, ya que
2
on.
necesariamente x 108 61 (mod 13 ) es la otra soluci
Nos queda ver el caso de restos cuadraticos modulo una potencia de 2, que estudiamos
a continuaci
on:
Teorema 5.19. Sea a un n
umero impar. Entonces:
(i) La congruencia x2 a (mod 2) tiene siempre solucion.
(ii) La congruencia x2 a (mod 4) tiene solucion si y s
olo si a 1 (mod 4).
(iii) Si k 3, la congruencia x2 a (mod 2k ) tiene solucion si y s
olo si a 1 (mod 8).
Ademas, en este caso, la congruencia tiene exactamente dos soluciones modulo 2k1
Demostraci
on: La parte (i) es obvia, y la parte (ii) sigue del hecho de que el cuadrado de
cualquier n
umero impar es de la forma 4k + 1. Para parte (iii) observamos que, como el
cuadrado de un n
umero impar es siempre de la forma 8k + 1, entonces si la congruencia
2
k
x a (mod 2 ) tiene solucion, entonces a 1 (mod 8). Demostraremos el recproco
por inducci
on sobre k, siendo el caso k = 3 trivial (las soluciones m
odulo 4 son 1 y 3).
2
k
Supongamos entonces que x a (mod 2 ) tiene solucion, es decir, que existe x0 Z tal
53

que x20 = a + 2k b para alg


un b Z. Veamos que existe una solucion de x2 a (mod 2k+1 )
de la forma x = x0 + 2k1 y. En efecto, como
(x0 + 2k1 y)2 = x20 + 2k x0 y + 22k2 y 2 = a + 2k (b + x0 y) + 22k2 y 2
y x0 es impar, si tomamos y de la misma paridad que b tendremos que x0 + 2k1 y es
solucion de x2 a (mod 2k+1 ).
on de x2 a (mod 2k ), entonces para cualquier otra
Ademas, si x0 es una soluci
soluci
on x se verica
2k |x2 x20 = (x + x0 )(x x0 )
y, como el maximo com
un divisor de x + x0 y x x0 es divisible por 2 pero no por 4, se
sigue que x x0 (mod 2k1 ) o x x0 (mod 2k1 ).

54

6. Ecuaciones diof
anticas
Se llama ecuaci
on diof
antica a una ecuaci
on cuyas variables son n
umeros enteros. La
n
n
mas conocida es la llamada ecuaci
on de Fermat, que es x + y = z n . Fermat aseguro
que, si n 3, las u
nicas posibles soluciones son aquellas en que alguno de los x, y, z es

cero. Este resultado, conocido como Ultimo


Teorema de Fermat, ha sido demostrado s
olo
recientemente por Wiles. En esta seccion estudiaremos los casos mas sencillos de esta
ecuacion, as como otras ecuaciones diofanticas similares.

Empezamos con el caso n = 2, que no entra dentro del Ultimo


Teorema de Fermat, ya
que en este caso s que existe solucion, y de hecho innitas, como demostraremos enseguida,
viendo que estructura tienen.
Denici
on. Se llama terna pitag
orica al conjunto de tres enteros x, y, z tales que x2 +y 2 =
orica, por el
z 2 . El nombre viene dado porque, si x, y, z > 0 forman una terna pitag
teorema de Pit
agoras existe un tri
angulo rect
angulo que tiene a x e y como longitudes de
sus catetos y a z como longitud de la hipotenusa. Un tri
angulo as (es decir, rectangulo
con las longitudes de los lados enteras) se llama tri
angulo pitag
orico.
Lema 6.1. Sea x, y, z una terna pitag
orica y d = mcd(x, y, z). Entonces:
(i) d = mcd(x, y) = mcd(x, z) = mcd(y, z).
(ii) Si escribimos x = dx , y = dy , z = dz , entonces x , y , z es una terna pitag
orica.
Demostraci
on: De (i) demostraremos solo d = mcd(x, y), siendo identicas las demostraciones de las otras igualdades. Sea d = mcd(x, y). Es evidente que mcd(x, y, z) divide a
d, por lo que basta demostrar que d divide a mcd(x, y, z). Para ello, observamos que d|x
y d|y implica d2 |x2 y d2 |y 2 . Por tanto, d2 |x2 + y 2 = z 2 , de donde se deduce d|z (por el
Teorema 1.18). De aqu se concluye d| mcd(x, y, z).
La parte (ii) es inmediata.

Denici
on. Una terna pitag
orica se dice que es primitiva si mcd(x, y, z) = 1, o equivalentemente cualquiera de los mcd(x, y), mcd(x, z), mcd(y, z) es 1.
Lema 6.2. Sea x, y, z una terna pitag
orica primitiva. Entonces entre x, y, z hay exactamente un n
umero par, que es o bien x o bien y.
Demostraci
on: Es evidente que entre x, y, z hay exactamente un n
umero par. Si fuera z,
2 2
2
entonces xx, y seran impares, luego x , y 1 (mod 8), luego z = x2 + y 2 2 (mod 8),
lo que es absurdo, porque z 2 debe ser divisible por 4.
55

Teorema 6.3. Sea x, y, z una terna pitag


orica primitiva con x, y, z > 0 y x par. Entonces
existen s, t Z tales que 0 < t < s, mcd(s, t) = 1, s t (mod 2) y

x =2st

y =s2 t2

z =s2 + t2
Demostraci
on: Observamos en primer lugar que mcd(z +y, z y) = 2. En efecto, cualquier
divisor com
un de z + y y z y lo es tambien de su suma y su diferencia, es decir, de 2z y
2y Como mcd(y, z) = 1, se sigue el u
nico posible divisor com
un de z + y y z y es 2, que
efectivamente es un divisor, ya que y, z son impares. Podemos escribir entonces z + y = 2a
y z y = 2b, con a, b enteros tales que mcd(a, b) = 1. Por tanto
y =ab
z =a+b
Observese que, como y < x2 + y 2 = z, entonces b > 0 y, como y > 0, tambien a > b.
Por otra parte, como x es par, podemos escribir x = 2c. De la igualdad x2 = z 2 y 2 =
(z + y)(z y) se deduce entonces c2 = ab, luego, por el Corolario 1.17(ii), existen enteros
positivos s, t tales que a = s2 y b = t2 (y por tanto x = 2st, y = s2 t2 y z = s2 +t2 ). Como
mcd(a, b) = 1, necesariamente mcd(s, t) = 1 y, como a > b, sera s > t. Finalmente,como
ni y ni z son pares, necesariamente s y t tienen distinta paridad.
Teorema 6.4. La ecuacion x4 + y 4 = z 2 no tiene soluci
on entera positiva.
Demostraci
on: Sea S el conjunto de los enteros positivos z para los que existen x, y > 0
2
on al absurdo, que la ecuaci
on x4 +y 4 = z 2
tales que z = x4 +y 4 . Supongamos, por reducci
tiene solucion entera positiva. Esto quiere decir que el conjunto S es no vaco, y por el
principio del buen orden existir
a un elemento mnimo de S. Sea pues una soluci
on x, y, z
2 2
orica, que veremos
con S mnimo en S. Los enteros x , y , z forman entonces una terna pitag
2 2
a continuaci
on que es primitiva. En efecto, si mcd(x , y ) = 1, entonces mcd(x, y) = d > 1.
Si escribimos x = dx , y = dy , entonces d4 |x4 + y 4 = z 2 , luego d2 |z y podremos escribir
z = d2 z , con z Z1 . Entonces, haciendo esas sustituciones en la igualdad x4 + y 4 = z 2
queda d4 x 4 + d4 y 4 = d4 z 2 , de donde se deduce x 4 + y 4 = z 2 . Por tanto, z S, lo que
es imposible porque z < dz = z y z era mnimo en S.
Sin perdida de generalidad, podemos suponer que x2 es par e y 2 , z impares. Por el
Teoorema 6.3, se tendra que existen s, t enteros positivos primos entre s y de distinta
paridad tales que
2
x =2st

y 2 =s2 t2

z =s2 + t2
56

En particular, t2 +y 2 = s2 , luego t, y, s es tambien una terna pitag


orica primitiva y adem
as
necesariamente s es impar y t es par. Usando de nuevo el Teorema 6.3, concluimos que
podemos encontrar u, v primos entre s y de distinta paridad tales que

t =2uv

y =u2 v 2

s =u2 + v 2
Como s es impar, mcd(2t, s) = 1, luego de la igualdad x2 = (2t)s y del Corolario 1.17(ii)
deducimos que existen t , s Z1 tales que
2t =(2t )2
s =s 2
Finalmente, haciendo la sustituci
on t = 2t 2 en la igualdad t = 2uv llegamos a t 2 = uv, y
usando de nuevo el Corolario 1.17(ii) y mcd(u, v) = 1, obtenemos que existen u , v Z1
tales que
u =u 2
v =v 2
Por tanto, la igualdad s = u2 + v 2 se convierte en s 2 = u 4 + v 4 , luego s S. La
desigualdad
s s 2 = s s2 < s2 + t2 = z
produce entonces una contradicci
on con el hecho de que z era mnimo en S.

Corolario 6.5. El Ultimo


Teorema de Fermat es cierto para n = 4.
on entera x =
Demostraci
on: En efecto, si la ecuacion x4 + y 4 = z 4 tuviera una soluci
2
on entera
a, y = b, z = c con abc = 0, entonces x = |a|, y = |b|, z = c sera una soluci
4
4
2
positiva de x + y = z , lo que es imposible por el Teorema 6.4.

Observaci
on 6.6. El corolario anterior reduce el Ultimo
Teorema de Fermat al caso en

que el exponente n es un n
umero primo. En efecto, supongamos demostrado el Ultimo
Teorema de Fermat para exponentes primos. Entonces, para cada n > 2, o bien n tiene un
divisor primo impar o es una potencia de 2. En el primer caso, si n = pk, con p > 2 primo,
entonces la ecuacion de Fermat se puede escribir como (xk )p + (y k )p = (z k )p , que no tiene
soluci
on no trivial, porque por hip
otesis xp + y p = z p no la tiene. Si, en cambio, n es una
potencia de 2, como n > 2, necesariamente se puede escribir n = 4k (ademas, k sera una
potencia de 2, aunque esto no nos interesa). En este caso, se puede escribir la ecuacion
como (xk )4 + (y k )4 = (z k )4 , pero el Corolario 6.5 implica que entonces xk y k z k = 0, es
decir, xyz = 0.
Veamos otra ecuacion diof
antica mas restrictiva que la de Fermat de grado cuatro:
57

Teorema 6.7. La ecuacion x4 + y 2 = z 4 no tiene soluci


on entera positiva.
Demostraci
on: Como en el Teorema 6.4, suponemos por reduccion al absurdo que exista
soluci
on y tomamos aquella con z mnimo, lo que implicar
a en particular que x2 , y, z 2 es
una terna pitag
orica primitiva. Tenemos que distinguir ahora dos casos seg
un la paridad
de x e y.
Si x es impar e y es par se tendra, por el Teorema 6.3, que existen s, t Z tales que
2
x =s2 t2

y =2st

2
z =s2 + t2
y, multiplicando la primera y u
ltima igualdad se obtiene x2 z 2 = (s2 t2 )(s2 + t2 ) = s4 t4 ,
y en particular t4 + (xz)2 = s4 . Como s2 < s2 + t2 = z 2 , se concluye que s < z, lo que
contradice la minimalidad de z.
a
Nos queda entonces el caso en que x2 es par e y impar. Por el Teorema 6.3, se tendr
que existen s, t enteros positivos primos entre s y de distinta paridad tales que
x2 =2st
z 2 =s2 + t2
donde no decimos si y = s2 t2 o y = t2 s2 ; por tanto, s y t juegan un papel simetrico, y
podemos suponer que s es par y t impar. Como la igualdad s2 + t2 = z 2 nos dice que s, t, z
es una terna pitag
orica (primitiva porque mcd(s, t) = 1), por el Teorema 6.3, podemos
encontrar u, v primos entre s y de distinta paridad tales que

s =2uv

t =u2 v 2

z =u2 + v 2
Como t es impar, mcd(2s, t) = 1, luego de la igualdad x2 = (2s)t y del Corolario 1.17(ii)
deducimos que existen s , t Z1 tales que
2s =(2s )2
t =t 2
Finalmente, haciendo la sustituci
on s = 2s 2 en la igualdad s = 2uv llegamos a s 2 = uv, y
usando de nuevo el Corolario 1.17(ii) y mcd(u, v) = 1, obtenemos que existen u , v Z1
tales que
u =u 2
v =v 2
58

Por tanto, la igualdad t = u2 v 2 se convierte en v 4 + t 2 = u 4 , y claramente


u u 2 = u u 2 < u2 + v 2 = z
lo que contradice la minimalidad de z.
Corolario 6.8. El area de un tri
angulo pitag
orico no puede ser un cuadrado perfecto.
Demostraci
on: Si x, y, z son las longitudes de los lados de un tri
angulo pitag
orico, el area
1
del mismo es 2 xy, luego si es un cuadrado perfecto existir
a u Z1 tal que xy = 2u2 . Por
tanto tendremos
(x + y)2 = x2 + 2xy + y 2 = z 2 + 4u2
(x y)2 = x2 2xy + y 2 = z 2 4u2
y multiplicando llegamos a
(x2 y 2 )2 = z 4 (2u)4
que contradice el Teorema 6.7.
Observaci
on 6.9. No podemos seguir poniendo m
as restricciones a la ecuacion de Fermat
de grado cuatro sin llegar ya a soluciones no triviales. Observese que en los Teoremas 6.4
y 6.7 hemos cambiado un exponente 4 por un 2. Si cambi
aramos dos exponentes 4 por un
2
4
2
2, es decir, si consideraramos las ecuaciones x + y = z y x2 + y 2 = z 4 , ya tendramos
innitas soluciones. Basta mirar en la expresi
on de las ternas pitag
oricas primitivas dada
en el Teorema 6.3 para ver que se obtienen innitas soluciones de x2 + y 4 = z 2 si existen
innitas soluciones de

x =2st

y 2 =s2 t2

z =s2 + t2
lo que es cierto porque existen innitas soluciones de y 2 = s2 t2 . De la misma forma,
existir
an innitas soluciones de x2 + y 2 = z 4 si existen innitas soluciones de

x =2st

y =s2 t2

2
z =s2 + t2
lo que vuelve a ser cierto, porque basta encontrar innitas soluciones de z 2 = s2 + t2 .
Las soluciones de la ecuacion x2 + y 2 = z 2 se pueden interpretar desde otro punto
de vista: cu
ando un cuadrado perfecto z 2 se puede escribir como suma de dos cuadrados
59

x2 + y 2 ? O bien, cu
ando un cuadrado perfecto x2 se puede escribir como diferencia de dos
on queremos estudiar un problema m
as general: cuando
cuadrados z 2 y 2 ? A continuaci
un n
umero se puede escribir como suma o diferencia de dos cuadrados?. Empezamos con
el caso de la diferencia de cuadrados, que resulta sorprendentemente f
acil:
Teorema 6.10. Un n
umero n
n 2 (mod 4).

se puede escribir como n = x2 y 2 si y solo si

Demostraci
on: Supongamos primero que n = x2 y 2 . Como x2 , y 2 0, 1 (mod 4), se
sigue que n = x2 y 2 0, 1, 3 (mod 4). Recprocamente, supongamos que n 2 (mod 4).
Distinguimos dos casos:
n1 2
2
-Si n es impar, entonces escribimos n = ( n+1
2 ) ( 2 ) .
n4 2
2
-Si n es par, entonces es m
ultiplo de 4, y podemos escribir n = ( n+4
4 ) ( 4 ) .

El caso de la suma de dos cuadrados es mas complicado. Empezamos con una observacion que ser
a muy u
til:
Lema 6.11. El producto de dos sumas de dos cuadrados es tambien una suma de dos
cuadrados.
Demostraci
on: En efecto, basta observar que (x2 + y 2 )(z 2 + w2 ) = (xz + yw)2 + (xw yz)2 .

A la vista del resultado anterior, parece l


ogico estudiar primero que n
umeros primos
son sumas de dos cuadrados. En este caso, tenemos el siguiente primer resultado:
Proposici
on 6.12. Si p es un n
umero primo y x, y Z, entonces:

(i) p = x2 + y 2 si y solo si p|x2 + x2 y 0 < |x|, |y| < p.


(ii) Si p = x2 + y 2 = z 2 + w2 , entonces o bien |x| = |z|, |y| = |w| o bien |x| = |w|, |y| = |z|.
Demostraci
on: Para demostrar (i), supongamos primero p = x2 + y 2 , de donde se deduce
inmediatamente p|x2 + x2 . Ademas, no puede ser x = 0, ya que entonces p = y 2 , y no
podra ser un n
umero primo. De la misma forma, y = 0, por lo que |x|, |y| > 0. De aqu

alogamente, |y| < p.


se deduce tambien x2 < x2 + y 2 = p, luego |x| < p y, an

Recprocamente, supongamos p|x2 +x2 y 0 < |x|, |y| < p. En particular, x2 +y 2 = np

para alg
un n > 0. Por otra parte, np = x2 + y 2 < ( p)2 + ( p)2 = p + p = 2p, luego
n < 2, es decir, n = 1 y x2 + y 2 = p.
Para demostrar (ii), es claro que basta suponer x, y, z, w > 0. Tenemos entonces
(xz yw)(xwyz) = x2 zwxyz 2 xyw2 +y 2 zw = (x2 +y 2 )zwxy(z 2 +w2 ) = p(zwxy).
60


Como p es primo, entonces p|xz yw o p|xw yz. Por otra parte, por (i), x, y, z, w < p,
luego |xz yw|, |xw yz| < p, por lo que decir que xz yw o xw yz son divisibles por
p es lo mismo que decir que xw yz o xw yz son cero y, en cualquiera de los caso, se
tendr
a tambien zw xy = 0. Supongamos por ejemplo xw yz = 0. Entonces
zw2 = w(zw) = w(xy) = y(xw) = y(yz) = y 2 z
de donde se sigue y 2 = w2 y por tanto x2 = w2 , es decir, y = w y x = z. Cuando
xw yz = 0, el mismo argumento demuestro x = w, y = z.
A la vista de la parte (i) del resultado anterior, empezamos viendo cu
ando una suma
de dos cuadrados es divisible por un n
umero primo.
Lema 6.13. Sea p un n
umero primo y sean x, y n
umeros enteros no divisibles por p. Sea
2
olo si a2 1 (mod p). En
a Z tal que ax y (mod p). Entonces p|x + y 2 si y s
particular, un n
umero primo p 3 (mod 4) no puede dividir a la suma de dos cuadrados
no divisibles por p.
Demostraci
on: Para la primera parte, basta observar que x2 y 2 a2 x2 (mod p).
Como mcd(x, p) = 1, por la Proposici
on 2.1(vi) la congruencia anterior es equivalente a
a2 1 (mod p).
Para la segunda parte, supongamos por reducci
on al absurdo que p 3 (mod 4) y
2
p|x + y con x, y no divisibles por p. Como mcd(x, p) = 1, por la Proposici
on 2.9, existir
a
2
a Z tal que ax y (mod p). El Lema 6.13 implica entonces a 1 (mod p), y por el
Teorema 2.19 no puede ser p 3 (mod 4).
2

El resultado anterior y la Proposici


on 6.12(i) sugieren que necesitaremos el siguiente:
Lema 6.14 (Thue). Sea p un n
umero primo y a Z con mcd(a, p) = 1. Entonces existen

x, y Z tales que ax y (mod p) y 0 < |x|, |y| < p.


Demostraci
on: Consideramos el conjunto de expresiones de la forma ax y con 0 x, y

p. Como tenemos ([ p] + 1)2 > p posibilidades para los pares x, y, necesariamente


tendremos dos pares distintos x1 , y1 y x2 , y2 en esas condiciones tales que ax1 y1
ax2 y2 (mod p). Por tanto, x = x1 x2 , y = y1 y2 es la solucion buscada.
Teorema 6.15. Un n
umero primo p se puede expresar como suma de dos cuadrados si y
solo si p = 2 o p 1 (mod 4).
Demostraci
on: Suponemos en primer lugar que podemos escribir p = x2 + y 2 . Por la
Proposici
on 6.12(i), 0 < |x|, |y| < p, luego x, y no son divisibles por p. El Lema 6.13
implica entonces que p no es congruente con 3 modulo 4, luego p = 2 o p 1 (mod 4).
61

Recprocamente, si p = 2 o p 1 (mod 4), el Teorema 2.19 implica que existe a Z


tal que p|a2 + 1. Evidentemente, mcd(a, p) = 1, por lo que por el Lema de Thue existen

x, y Z tales que ax y (mod p) y 0 < |x|, |y| < p. La condici


on ax y (mod p)

2
2
implica, por el Lema 6.13, que p|x + y , y entonces la condicion 0 < |x|, |y| < p implica,
por el Lema 6.13, p = x2 + y 2 , como queramos.
Podemos caracterizar nalmente los enteros que son sumas de dos cuadrados:
Teorema 6.16. Un n
umero positivo es suma de dos cuadrados si y solo si, en su descomposicion en factores primos, todos los factores primos de la forma 4k + 3 aparecen con
exponente par.
Demostraci
on: Sea n = x2 + y 2 y llamemos d = mcd(x, y). Podemos entonces escribir
x = dx y y = dy con mcd(x , y ) = 1. Entonces n = d2 (x 2 + y 2 ). Cada factor primo p
de x 2 + y 2 , no puede dividir ni a x ni a y (si dividiera a uno, dividira a ambos), luego
por la Lema 6.13, no es p 3 (mod 4). Por tanto, los factores primos de n de la forma
4k + 3 estan todos en d2 , luego aparecen con exponente par.
Recprocamente, si n tiene todos sus factores primos de la forma 4k + 3 con exponente
par, se podr
a escribir n = d2 p1 . . . pr , donde p1 , . . . , pr son primos distintos que no son de
la forma 4k +3. Por el Teorema 6.15, cada pi es suma de dos cuadrados, y por el Lema 6.11
su producto tambien lo es, es decir, p1 . . . pr = x2 + y 2 . De aqu se sigue n = (dx)2 + (dy)2 .

Observaci
on 6.17. Notese que, si n no es primo, no se tiene la unicidad de la descomposicion en suma de dos cuadrados que obtuvimos en la Proposici
on 6.12(ii). En efecto,
2
2
consideremos n = 65 = 5 13. Tenemos entonces 5 = 2 + 1 y 13 = 32 + 22 . Aplicando
el Lema 6.11 con x = 2, y = 1, z = 3, w = 2, obtenemos 65 = 82 + 12 , mientras que, si
aplicamos el Lema 6.11 con x = 2, y = 1, z = 2, w = 3, obtendremos 65 = 72 + 42 .
Ya que hay muchos n
umeros positivos que no se pueden escribir como suma de dos
cuadrados, cabe plantearse ahora al pregunta: podr
a escribirse todo n
umero positivo
como suma de tres cuadrados? Inmediatamente encontramos una respuesta negativa:
Teorema 6.18. Ning
un n
umero de la forma 4n (8k + 7) se puede escribir como suma de
tres cuadrados.
Demostraci
on: Lo demostramos por inducci
on sobre n. Si n = 0, tenemos que ver que no
2
2
2
se puede escribir 8k +7 = x +y +z . Esto es evidente, ya que x2 , y 2 , z 2 0, 1, 4 (mod 8).
62

Supongamos ahora demostrado el resultado para n y veamos que es cierto para n + 1.


Supongamos, por reducci
on al absurdo, que podemos escribir 4n+1 (8k + 7) = x2 + y 2 +
z 2 . Como 4n+1 0 (mod 4) y x2 , y 2 , z 2 0, 1 (mod 4), se sigue que necesariamente
a2 , b2 , c2 0 (mod 4), es decir, x, y, z son pares. Escribimos entonces x = 2x , y = 2y y
a 4n (8k + 7) = x 2 + y 2 + z 2 , lo que contradice la hip
otesis de
z = 2z , por lo que se tendr
inducci
on.
Observaci
on 6.19. En realidad, se puede demostrar que el teorema anterior caracteriza
a los n
umeros que no son sumas de tres cuadrados. Sin embargo, la demostraci
on no es
f
acil (por lo que no la haremos). Esto se debe a que, contrariamente a lo que ocurre para
la suma de dos cuadrados (ver el Lema 6.11) el hecho de ser suma de tres cuadrados no se
respeta por productos. Por ejemplo, 14 = 32 + 22 + 12 y 18 = 42 + 12 + 12 , mientras que
14 18 = 4(8 7 + 7) no es suma de tres cuadrados por el Teorema 6.18.
Ya que con tres cuadrados no se puede, podemos seguir pregunt
andonos ahora: Y ser
a
cierto que todo n
umero entero positivo se puede escribir como suma de cuatro cuadrados?
La respuesta ahora sera positiva, y se basa en que, en este caso ya es cierto el analogo del
Lema 6.11, que es una simple cuenta que dejamos como ejercicio:
Ejercicio 6.20. Comprobar que se verica la igualdad
(a2 + b2 + c2 + d2 )(x2 + y 2 + z 2 + w2 ) = r2 + s2 + t2 + u2
donde

r = ax + by + cz + dw

s = bx ay + dz cw

t = cx dy az + bw

u = dx + cy bz aw

A la vista de este resultado, bastar


a ver que todo n
umero primo es suma de cuatro
cuadrados. Como en el caso de dos cuadrados, empezamos de momento con propiedades
sobre los divisores de distintas sumas de cuadrados:
Lema 6.21. Si p 3 (mod 4) es un n
umero primo, entonces existen x, y
2
2
p|x + y + 1.

Z tales que

Demostraci
on: Por las partes (iii) y (iv) de la Proposici
on 5.3, tenemos respectivamente
1
p

=1y

que

a1
p

p1
p

=1y

a
p

1
p

= 1. Podremos encontrar entonces a {2, 3, . . . , p 1} tal

= 1 y, en particular,
a
p

1
p

a
p
63

= (1)(1) = 1.

Por tanto, existen a 1 y a son restos cuadraticos modulo p, es decir, existen x, y


tales que x2 a 1 (mod p) y y 2 a (mod p). Sumando, tendremos

x2 + y 2 (a 1) + (a) = 1 (mod p),


es decir, p|x2 + y 2 + 1.
Lema 6.22. Sea m un divisor impar de una suma x2 + y 2 + z 2 + w2 . Entonces existen
a, b, c, d Z tales que

x a (mod m)

y b (mod m)

z c (mod m)

w d (mod m)
y a2 + b2 + c2 + d2 = mm , con m < m.
Demostraci
on: Sea r el resto de la divisi
on de x entre m. Si r < m
2 tomamos a = r, y si
m
r > 2 tomamos a = r m (hace falta que m sea impar para que no se pueda dar el caso
m
r= m
2 . Entonces x a (mod m) y |a| < 2 . Construimos de la misma forma b, c, d, es
decir, congruentes respectivamente con y, z, w modulo m y de valor absoluto menor que
m
2 . Por tanto, tendremos
a2 + b2 + c2 + d2 x2 + y 2 + z 2 + w2 0 (mod m)
luego existir
a m Z tal que a2 + b2 + c2 + d2 = mm . Ademas,
mm = a2 + b2 + c2 + d2 <

m
2

m
2

m
2

m
2

= m2

por lo que m < m.


Finalmente, podemos demostrar ya el resultado que busc
abamos:
Teorema 6.23. Todo n
umero natural se puede escribir como suma de cuatro cuadrados
(no necesariamente distintos de cero).
Demostraci
on: Por el Ejercicio 6.20, basta demostrar que cada n
umero primo p es suma
de cuatro cuadrados. Si p = 2 o p 1 (mod 4) ya sabemos que p se escribe como suma de
incluso s
olo dos cuadrados, as que basta demostrar el caso p 3 (mod 4). Por el Lema
6.21, sabemos que p divide a la suma de tres cuadrados de enteros no todos nulos, luego en
particular un m
ultiplo de p se puede poner como la suma de cuatro cuadrados de enteros
no todos nulos. Sea kp el m
ultiplo m
as peque
no de p que se puede escribir como suma de
64

cuatro cuadrados de enteros positivos no todos nulos. Por el Lema 6.22 tomando m = p,
se tiene k < p, y lo que queremos ver es k = 1.
Veamos primero que k es impar. Escribimos
kp = x2 + y 2 + z 2 + w2 .
Si k fuera par, entonces entre x, y, z, w hay una cantidad par de n
umeros pares y una
cantidad par de impares. En cualquier caso, reorden
andolos, podemos suponer que x e y
tienen la misma paridad y que z y w tienen la misma paridad. Entonces tendremos
k
p=
2

x+y
2

xy
2

z+w
2

zw
2

lo que contradice la minimalidad de k, lo que implica que k es impar.


Supongamos, por reducci
on al absurdo, que k > 1. Aplicando ahora el Lema 6.22
tomando m = k, existir
an a, b, c, d Z tales que

x a (mod k)

y b (mod k)

z c (mod k)

w d (mod k)
y a2 + b2 + c2 + d2 = nk con n < k. Si fuera n = 0, entonces a = b = c = d = 0, luego k
dividira a x, y, z, w, as que k 2 dividira a x2 + y 2 + z 2 + w2 = kp, es decir, k dividira a
p, lo que contradice el hecho de que 1 < k < p. Por tanto, 0 < n < k. Usando la igualdad
del Ejercicio 6.20 tendremos
k 2 np = (a2 + b2 + c2 + d2 )(x2 + y 2 + z 2 + w2 ) = r2 + s2 + t2 + u2
donde

r = ax + by + cz + dw

s = bx ay + dz cw

t = cx dy az + bw

u = dx + cy bz aw

x2 + y 2 + z 2 + w2 0 (mod k)
ba ab + dc cd = 0 (mod k)
ca db ac + bd = 0 (mod k)
da + cb bc ad = 0 (mod k)

es decir, r, s, t, u son divisibles por k. Se tiene entonces


np =

r
k

s
k

t
k

u
k

lo que de nuevo contradice la minimalidad de k, ya que n < k. Por tanto, k = 1, como


queramos.
65

Observaci
on 6.24. Para sumas de potencias superiores, se tiene el llamado problema
umero g(n) tal que cada
de Waring. En concreto, jado n Z1 , cual es el mnimo n
n
umero entero positivo es suma de g(n) potencias n-esimas de n
umeros enteros? En este
contexto, el Teorema 6.18 indica que g(2) 4 y el Teorema 6.23 indica que g(2) 4, por
lo que g(2) = 4. Para n 3 el problema es mucho mas complicado, ya que no ocurre que
el producto sumas de g potencias n-esimas sea una suma de g potencias n-esimas (como
ocurre para n = 2 con g = 2 y g = 4). Hay adem
as una situaci
on nueva. Por ejemplo,
3
3
3
3
3
3
para n = 3 no es difcil ver que 23 = 2 + 2 + 1 + 1 + 1 + 1 + 13 + 13 + 13 es la menor
descomposicion en suma de cubos de 23, por lo que g(3) 9. Por otra parte, ha logrado
demostrarse (por L. Dickson) que cualquier entero positivo distinto de 23 y 239 puede
escribirse como suma de como mucho ocho cubos, y de la misma forma se sabe (por un
resultado debido a Linnik) que salvo una cantidad nita de casos, todo entero positivo es
suma de a lo sumo de siete cubos. Esto da lugar a otro problema, llamado el gran problema
de Waring (al anterior se le suele llamar por contraposici
on peque
no problema de Waring):
cual es el mnimo n
umero G(n) tal que cada n
umero entero positivo, salvo una cantidad
nita de casos, es suma de G(n) potencias n-esimas de n
umeros enteros? Mientras que
para n = 2 sigue siendo g(2) = G(2) = 4, lo anterior para n = 3 se traduce en g(3) = 9,
G(3) 7. Es un problema abierto cu
al es el valor de G(3) (se sospecha G(3) = 4, pero
solo se sabe 4 G(3) 7) y en general de G(n) para n 3. Respecto a g(n), se ha
demostrado recientemente que g(4) = 19, g(5) = 37 y g(n) = [( 32 )n ] + 2n 2 si n 6 salvo
quiz
a una cantidad nita de valores de n.
Los Teoremas 6.10 y 6.16 pueden interpretarse como haber estudiado cu
ando las ecua2
2
2
2
on entera. El caso m
as sencillo
ciones de la forma x y = n o x + y = n tienen soluci
ha sido el de diferencia de dos cuadrados. Podemos complicar la ecuaci
on un poco m
as y
poner coecientes a la x y a la y, en cuyo caso las cosas se complican muchsimo. El primer
caso, aparentemente sencillo, sera la ecuacion x2 dy 2 = 1 con d entero positivo que no
sea un cuadrado perfecto. Esta ecuaci
on (llamada ecuaci
on de Pell, aunque en realidad
se debera atribuir a Fermat), no es f
acil en absoluto, aunque al menos se puede decir que
sus soluciones (caso de existir) tienen una buena estructura:
Teorema 6.25. Sea d un entero positivo que no sea un cuadrado perfecto y consideremos
la ecuacion x2 dy 2 = 1.
(i) Si x , y y x , y son soluciones enteras de la ecuacion, entonces x x +dy y , x y +x y
es tambien solucion de la ecuaci
on.

(ii) Si existe una soluci


on entera positiva x1 , y1 tal que x1 + y1 d es mnimo entre todas
las soluciones enteras positivas de la ecuacion, entonces las soluciones enteras positivas

de la ecuacion son los pares de la forma xn , yn donde xn + yn d = (x1 + y1 d)n .


66

Demostraci
on: Observese que
(x + y

d)(x + y

d) = (x x + dy y ) + (x y + x y ) d

d) = (x x + dy y ) (x y + x y ) d

Como las soluciones de la ecuacion son los enteros que verican (x + y d)(x y d) = 1,
multiplicando las dos igualdades anteriores se obtiene (i). De la misma forma se obtiene
on. Lo que falta ver es que son las u
nicas soluciones.
que los pares xn , yn de (ii) son soluci
(x y

d)(x y

Supongamos, por reducci


on al absurdo, que tenemos otra soluci
on entera positiva

x , y que no es ninguna de las xn , yn . Por la minimalidad de x1 , y1 , se tendr


a x1 + y1 d <

x +y d. Ademas como la sucesion (x1 +y1 d)n es monotona creciente con lmite innito
se tendr
a que existe n Z1 tal que

(x1 + y1 d)n < x + y d < (x1 + y1 d)n+1 .

Multiplicando por (x1 y1 d)n (que es positivo porque su producto con (x1 + y1 d)n es
1) se tiene

1 < (x + y d)(x1 y1 d)n < x1 + y1 d.


Ahora bien, por (i), si escribimos
(x + y

d)(x1 y1 d)n = (x + y d)(xn yn d) = x + y d

se tiene que x , y es una solucion de la ecuacion. Para encontrar el absurdo que buscamos

basta ver que esta u


ltima soluci
on es positiva. Ahora bien, como x + y d > 1 y

(x + y d)(x y d) = 1, entonces 0 < x y d < 1. Por tanto,


2x = (x + y
2y

d = (x + y

d) + (x y

d) (x y

d) > 1 + 0 > 0

d) > 1 1 > 0

lo que termina la demostraci


on.
Esta demostracion indica lo que es una de las tecnicas fundamentales de la Teora
Algebraica de N
umeros: ampliar el conjunto de los enteros a expresiones, por ejemplo, de

la forma a + b d. El problema principal de esta tecnica es que, salvo para valores muy
concretos de d, no se verica la factorizaci
on u
nica que tenemos para los enteros (y cuando
se verica no es de forma tan f
acil). Para terminar de estudiar la ecuaci
on de Pell (y en particular la existencia de la soluci
on x1 , y1 que necesita el teorema anterior) desarrollaremos
en la siguiente seccion toda una tecnica nueva, la de las fracciones continuas.
67

7. Fracciones continuas
Cuando pasamos de los n
umeros enteros a los racionales, la escritura que solemos
hacer de ellos, aparte de como fracciones, es a partir de su notaci
on decimal. Esta escritura
decimal presenta dos inconvenientes. Uno es que depende de una elecci
on de escritura en
base 10, que a priori es arbitraria. El segundo inconveniente es que las expresiones con
decimales suelen ser innitas, aunque al menos son peri
odicas y se pueden representar de
forma nita.
Ejemplo 7.1. Veamos una tercera representacion de un n
umero racional a partir del
a
algoritmo de Euclides. Si tenemos el n
umero b , escribimos
a = ba0 + r1
b = r1 a1 + r2
r1 = r2 a2 + r3
..
.

0 < r1 < b
0 < r2 < r1
0 < r3 < r2

rn2 = rn1 an1 + rn


rn1 = rn an

0 < rn < rn1

donde rn = mcd(a, b). Ademas, a1 , . . . , ar > 0 porque son cocientes de n


umeros positivos
uno mayor que el otro. Podemos entonces escribir:
a
r1
= a0 +
b
b
r2
b
= a1 +
r1
r1
r1
r3
= a2 +
r2
r2
..
.
rn
rn2
= an1 +
rn1
rn1
rn1
= an
rn
(es decir, a0 , a1 , . . . , ar son las partes enteras de
r1
1
a
= a0 +
= a0 +
b
b
a1 +

r2
r1

= a0 +

rn1
a b
b , r1 , . . . , rn )

1
a1 +

1
r
a2 + r3
2

68

y, poniendo todo junto,

= . . . = a0 +

1
a1 +

1
a2 +

..

1
an1 + 1
an

Ejemplo 7.2. Podramos intentar ahora repetir un proceso parecido para cualquier
n
umero: dado un n
umero real = 0, vamos escribiendo
0 := = [0 ] + (0 [0 ]) = a0 + (0 a0 )
1 :=

1
= [1 ] + (1 [1 ]) = a1 + (1 a1 )
0 a0
..
.

es decir, denimos por recurrencia, a partir de 0 = , los n


umeros ak y k mediante:
ak = [k ]
k+1 =

1
.
k ak

Tendremos entonces
= a0 +

1
a1 +

1
a2 +

..

1
ak1 +

ak +

1
1
k+1

Observese que, como ak k < ak + 1, necesariamente 0 k ak < 1, por lo que, si


k = ak (es decir, si k no es entero), se tendra k+1 > 1, por lo que ak 1 si k 1
(aunque a0 puede ser cero o incluso negativo). Si es racional, el Ejemplo 7.1 indica que el
proceso termina (lo que ocurre cuando llegamos a alg
un k entero y, por tanto ak = k ),
mientras que si es irracional, entonces cada k es tambien irracional, y por tanto el

proceso no termina nunca. Por ejemplo, si tomamos 2, tendremos entonces

0 = 2 = 1 414213562 . . . = 1 + 0 414213562 . . . = 1 + ( 2 1)

1
= 2 + 1 = 2 414213562 . . . = 2 + 0 414213562 . . . = 2 + ( 2 1)
21

y, a partir de aqu, 1 = 2 = . . . = 2 + 1, y por tanto a1 = a2 = . . . = 1, mientras


a0 = 1. Parecera entonces natural escribir
1 =

2=1+

1
2+

1
2+

2+

1
1
2+...

Denici
on. Se llama fracci
on continua a una expresi
on del tipo
[a0 ; a1 , a2 , . . .] := a0 +

69

1
a1 +

1
a2 +

a3 +

1
1
a4 +...

donde los n
umeros a1 , a2 , . . . (que pueden ser una cantidad nita o innita) son estrictamente positivos. Se llama fracci
on continua asociada a un n
umero real a la fracci
on
on concontinua [a0 ; a1 , a2 , . . .] denida como en el Ejemplo 7.2. Se dice que una fracci
umeros enteros (y por tanto a1 , a2 , . . . 1). Usaremos
tinua es simple si a0 , a1 , . . . son n
on continua innita
tambien la notaci
on [a0 ; a1 , . . . , ar , ar+1 , . . . , ar+s ] para indicar la fracci
peri
odica [a0 ; a1 , . . . , ar , ar+1 , . . . , ar+s , ar+1 , . . . , ar+s , ar+1 , . . . , ar+s , . . .].
Por lo que hemos visto, un n
umero racional se expresa siempre como una fraccion
continua simple nita, y es obvio que las fracciones continuas simples nitas representan
n
umeros racionales. Por otra parte, el metodo anterior sugiere que los n
umeros irracionales
tienen asociada una fracci
on continua innita. Para dar sentido a dicha expresi
on innita,
habr
a que verla como lmite de sus subfracciones innitas. M
as precisamente:
Denici
on. Se llama convergente k-esimo de una fracci
on continua simple [a0 ; a1 , a2 , . . .]
(nita o innita) a Ck = [a0 ; a1 , a2 , . . . , ak ].
on continua (no necesariamente simple o
Teorema 7.3. Sea [a0 ; a1 , a2 , . . .] una fracci
nita). Denimos recursivamente dos sucesiones de n
umeros pk y qk con k 2 mediante

q =1
p2 =0

p1 =1
q1 =0

pk =ak pk1 + pk2


qk =ak qk1 + qk2
Entonces:
(i) Para cada k 0 se tiene Ck =

pk
qk .

(ii) Para cada k 2 se tiene pk+1 qk pk qk+1 = (1)k .


Demostraci
on: Demostraremos (i) por inducci
on sobre k. Para k = 0, se tiene p0 = a0 ,
p0
q0 = 1, luego q0 = a0 = [a0 ] = C0 .
Supongamos ahora que sabemos que la f
ormula es cierta para convergentes k-esimos
y veamos que es cierta para los (k + 1)-esimos. Para ello consideramos la fraccion continua
[a0 ; a1 , a2 , . . . , ak ] := [a0 ; a1 , a2 , . . . , ak +

1
ak+1

],

que claramente coincide con Ck+1 . Por otra parte, es claro que si i k 1 entonces pi = pi
y qi = qi , mientras que para i = k se tiene
pk = ak pk1 + pk2 = (ak +
qk = ak qk1 + qk2 = (ak +

1
ak+1
1
ak+1

)pk1 + pk2 = ak pk1 + pk2 +

pk1
pk1
= pk +
ak+1
ak+1

)qk1 + qk2 = ak qk1 + qk2 +

qk1
qk1
= qk +
ak+1
ak+1

70

luego, por hip


otesis de induccion,
Ck+1

pk +
p
= Ck = k =
qk
qk +

pk1
ak+1
qk1
ak+1

ak+1 pk + pk1
pk+1
=
ak+1 qk + qk1
qk+1

Demostramos (ii) tambien por inducci


on sobre k, siendo trivial el caso k = 2.
Supuesta cierta la f
ormula para k, se tiene
pk+2 qk+1 pk+1 qk+2 = (ak+2 pk+1 + pk )qk+1 pk+1 (ak+2 qk+1 + qk ) = pk qk+1 pk+1 qk =
= (pk+1 qk pk qk+1 ) = (1)k = (1)(k+1) .

Ejemplo 7.4. La parte (ii) del teorema anterior implica, si la fracci


on continua es simple,
que pk , qk son enteros primos entre s (usando por ejemplo el Teorema 1.8). De hecho, puede
usarse esta parte para obtener explcitamente una combinaci
on lineal de dos enteros que
de su maximo com
un divisor. Por ejemplo, consideremos los enteros a = 34, b = 14. Si
aplicamos el algoritmo de Euclides podremos escribir
34 = 2 14 + 6
14 = 2 6 + 2
6=32+0
de donde sacamos

34
= [2; 2, 3].
14

Las sucesiones pk , qk valen entonces


p0 = 2, p1 = 5, p2 = 17
q0 = 1, q1 = 2, q2 = 7
lo que nos da 34
14 = C2 =
parte nos da la relaci
on

17
7 .

Esto indica, por una parte, que mcd(34, 14) = 2, y por otra
2 17 5 7 = 1,

que, cambiando de signo y multiplicada por el m


aximo com
un divisor nos da
5 14 2 34 = 2
71

Lema 7.5. Dada una fracci


on continua, se tiene, para todo k 0,:
(i) Ck+1 Ck =
(ii) Ck+2 Ck =

(1)k
qk+1 qk .
(1)k 1
qk+1 ( qk

1
qk+2 ).

(iii) Si la fracci
on es simple, 0 < q1 < q2 < . . ..
Demostraci
on: Por el Teorema 7.3, tenemos
Ck+1 Ck =

pk+1
pk
pk+1 qk pk qk+1
(1)k

=
=
,
qk+1
qk
qk+1 qk
qk+1 qk

lo que demuestra (i). Utilizando dos veces (i) tendremos


Ck+2 Ck = (Ck+2 Ck+1 ) + (Ck+1 Ck ) =

(1)k+1
(1)k
(1)k
+
=
qk+2 qk+1 qk+1 qk
qk+1

1
1

qk
qk+2

lo que demuestra (ii).


Para la parte (iii), se tiene q1 = a1 > 0, luego basta demostrar qk1 < qk para todo
k 2. Lo hacemos por inducci
on sobre k. Si k = 2, tenemos q2 = a2 q1 + q0 = a2 q1 + 1 >
a2 q1 . Como a2 1 (ya que a2 > 0 y a2 es entero por ser la fraccion simple), se sigue
on sigue los mismos pasos. Si suponemos que se verica
q2 > q1 . El paso de inducci
q1 < q2 < . . . < qk1 < qk , entonces en particular qk1 , qk > 0. Como de nuevo se tiene
ak+1 1 por ser la fracci
on simple, se sigue:
qk+1 = ak+1 qk + qk1 qk + qk1 > qk .
Teorema 7.6. Dada una fracci
on continua simple, se tiene, para todo k 0,
C0 < C2 < C4 < . . . < C2k < C2k+1 < . . . < C3 < C1 .
Ademas, si la fraccion continua es innita, existe el lmite de la sucesion {Ck }, que es un
n
umero irracional que verica C2k < < C2k+1 para todo k.
k

1
1
as qk+1 > 0
Demostraci
on: Por el Lema 7.5, se tiene Ck+2 Ck = (1)
qk+1 ( qk qk+2 ) y adem
y qk < qk+2 , luego Ck+2 Ck es positivo si k es par y negativo si k es impar, por lo que
los convergentes pares son crecientes y los impares son decrecientes. Ademas Ck+1 Ck =
(1)k
qk+1 qk , que es positivo si k es par y negativo si k es impar. Por tanto, un convergente
impar es siempre mayor que su convergente anterior par.

Tenemos entonces la sucesion creciente C0 < C2 < C4 < . . ., que esta acotada, luego
tiene un lmite , y la sucesion decreciente C1 > C3 > C5 > . . ., que tambien esta acotada,
luego tiene lmite . Ademas, para todo k se tiene
< C2k+1 C2k =
72

(1)2k
,
q2k+1 q2k

que es un valor que tiende a cero (por ser qk una sucesion creciente de n
umeros enteros),
luego = .
Veamos nalmente que es irracional. Supongamos, por reducci
on al absurdo, que
a
fuera = b , con a, b enteros (y b > 0). Entonces, como para cada k se tiene que esta
entre Ck y Ck+1 (quien sea el mayor de los dos depende de la paridad de k), se tiene
1
a
.
0 < | Ck | < |Ck+1 Ck | =
b
qk+1 qk
Multiplicando por bqk , se obtiene
b
0 < |aqk bpk | <
,
qk+1
lo que es absurdo, porque aqk bpk es un entero y si k es sucientemente grande se tendra
b
qk+1 < 1 (ya que qk+1 tiende a innito).
Denici
on. Se llama valos de una fracci
on continua simple innita al lmite dado por
el Teorema 7.6.
Ejemplo 7.7. Veamos ahora un ejemplo de c
alculo del valor de una fracci
on continua
innita. Tomemos, por ejemplo, [1; 1]. Las sucesiones pk , qk verican
p1 = 1, p0 = 1, pk = pk1 + pk2
q0 = 1, q1 = 1, qk = qk1 + qk2 ,
por lo que pk = uk+2 y qk = uk+1 (donde un es el n-esimo n
umero de Fibonacci; ver
uk+2
ormula del Ejercicio 0.3 para el
Ejercicio 0.3). Se tiene entonces Ck = uk+1 . Usando la f
n-esimo n
umero de
Fibonacci, es un simple ejercicio de An
alisis Matematico demostrar
uk+2
1
5
odicas, hay un truco
que lim uk+1 = 2 + 2 . Sin embargo, para fracciones continuas peri
mas facil para calcular su valor. Como el Teorema 7.6 nos asegura que [1; 1] tiene un valor
, se tendr
a la relacion:
1
1
=1+
=1+ ,
1

1 + 1+ 1
1+

1
1+...

por lo que debiera ser 2 1 = 0, es decir, =


la soluci
on positiva).

1
2

5
2

(obviamente, debemos tomar

Tenemos entonces un modo de, a partir de un n


umero irracional, conseguir una fracci
on
continua simple innita; y tenemos tambien un modo de, a partir de una fracci
on continua
simple innita, darle un valor irracional. Una pregunta natural es si ambos procesos son
inversos el uno del otro. Por ejemplo,
se deja como ejercicio al lector comprobar que

1
5
la fracci
on continua asociada a 2 + 2 (ver el Ejemplo 7.7) es precisamente [1; 1]; de la

misma forma, el valor de la fracci


on continua [1; 2] (ver el Ejemplo 7.2) es 2. El resultado
(armativo) general es el siguiente:
73

Teorema 7.8. Cada n


umero irracional es el valor de una y s
olo una fracci
on continua
innita simple, que es precisamente la fracci
on continua asociada a .
Demostraci
on: Supongamos primero que un n
umero irracional es el valor de las fracciones
continuas simples [a0 ; a1 , a2 , . . .] y [a0 ; a1 , a2 , . . .]. Entonces, por el Teorema 7.6, tendremos
C0 < < C1 , es decir a0 < < a0 + a11 . Como a1 1, se tiene entonces a0 = [], y de la
misma forma a0 = [], por lo que a0 = a0 . Como se tiene
a0 +

1
1
= [a0 ; a1 , a2 , . . .] = [a0 ; a1 , a2 , . . .] = a0 +
[a1 ; a2 , a3 , . . .]
[a1 ; a2 , a3 , . . .]

se deduce [a1 ; a2 , a3 , . . .] = [a1 ; a2 , a3 , . . .], y por un argumento de recurrencia se sigue


ak = ak para todo k.
Recprocamente, dado un n
umero irracional , sea [a0 ; a1 , a2 , . . .] su fracci
on continua
asociada (tomaremos la notacion del Ejemplo 7.2). Es claro que, para todo k, se tiene =
[a0 ; a1 , a2 , . . . , ak , k+1 ]. Entonces el convergente (k + 1)-esimo de esta fraccion continua
nita es , mientras que el convergente k-esimo es claramente el convergente k-esimo Ck
de [a0 ; a1 , a2 , . . .]. Por el Lema 7.5 aplicado a [a0 ; a1 , a2 , . . . , ak , k+1 ], se tendr
a entonces
Ck =

(1)k
,
(k+1 qk + qk1 )qk

donde hemos usado el Teorema 7.3 para calcular el denominador del convergente (k + 1)esimo. Usando ahora las desigualdades k+1 > ak+1 y qk+1 > qk se tiene
| Ck | =

1
1
1
1
<
=
< 2.
(k+1 qk + qk1 )qk
(ak+1 qk + qk1 )qk
qk+1 qk
qk

Como qk tiende a innito, se tiene que = lim Ck .

Ejercicio 7.9. Con la ayuda de una calculadora, comprobar la validez de la f


ormula (al
menos en sus primeros terminos):
e = [2; 1, 2, 1, 1, 4, 1, 1, 6, 1, 1, 8, . . .]
y utilizar la misma para calcular las diez primeras cifras decimales de e.

Observaci
on 7.10. La demostracion tanto del Lema 7.5 como del Teorema 7.8 nos
da el grado de aproximaci
on de un convergente a un n
umero irracional , precisamente
pk
1
1
| qk | < qk+1 qk < q2 . Por otra parte, si una considera una soluci
on positiva x = p, y = q
k

74

de la ecuacion de Pell x2 dy 2 = 1, se tiene p2 = 1 + q 2 d > q 2 d, de donde se deduce

p > q d. Por tanto,

1
1 (p q d)(p + q d)
1
1
1
p
< 2.

<
=
| d| = (p q d) =
q
q
q
2q
(p + q d)
q(p + q d)
2q 2 d

Cabe entonces preguntarse, ya que pq aproxima d como un convergente (incluso en principio a


un m
as), si eso no implicara que pq es realmente un convergente. Si miramos por

ejemplo el caso d = 2 del Ejemplo 7.2, en que demostramos 2 = [1; 2] tenemos los
siguientes valores:
p0 = 1, p1 = 3, p2 = 7, p3 = 17, . . .
q0 = 1, q1 = 2, q2 = 5, q3 = 12, . . .
Por tanto (p0 , q0 ) y (p2 , q2 ) son soluciones de la ecuacion x2 2y 2 = 1, mientras que (p1 , q1 )
y (p3 , q3 ) no son soluciones. De hecho, veremos que las soluciones son convergentes, pero
no todos los convergentes son soluciones.
Para ver que un n
umero racional que aproxima mucho a un irracional es un convergente
necesitaremos el siguiente lema previo.
Lema 7.11. Sea un n
umero irracional, sea pqkk el convergente k-esimo de su fraccion
continua y sea q un entero positivo tal que q < qk+1 . Entonces, para todo entero p se tiene
|pk qk | |p q|.
Demostraci
on: Observamos en primer lugar que existen x, y Z tales que
pk x + pk+1 y = p
qk x + qk+1 y = q.
En efecto, el determinante de la matriz de coecientes es, por el Teorema 7.3(ii),
pk
qk

pk+1
= pk qk+1 pk+1 qk = (1)k = 1
qk+1

luego existe una u


nica soluci
on del sistema anterior, que es ademas entera.
Observamos tambien que x = 0, ya que si fuera x = 0, entonces
0 = pq qp = p(yqk+1 ) q(ypk+1 ) = y(pqk+1 qpk+1 )
y como y = 0 (ya que q = 0), se sigue pqk+1 = qpk+1 . Ahora bien, mcd(pk+1 , qk+1 ) = 1,
luego qk+1 |q, lo que es absurdo porque q < qk+1 .
75

Comparemos ahora los signos de x e y. Si y < 0, entonces


qk x = q qk+1 y > 0
de donde se sigue x > 0. Si en cambio y > 0, entonces y 1 y se tiene
qk x = q qk+1 y < qk+1 qk+1 y = qk+1 (1 y) 0
y por tanto x < 0.
Usando lo anterior, tenemos
|p q| = |(pk x + pk+1 y) (qk x + qk+1 y)| = |x(pk qk ) + y(pk+1 + qk+1 )|.
Si supieramos que |x(pk qk ) + y(pk+1 + qk+1 )| |x(pk qk )|, se concluira inmediatamente, ya que
|x(pk qk )| = |x||pk qk | |pk qk |
(puesto que x = 0 y por tanto |x| 1). La desigualdad |x(pk qk ) + y(pk+1 + qk+1 )|
|x(pk qk )| es inmediata si y = 0, mientras que si y = 0 hay que demostrar que x(pk qk )
e y(pk+1 qk+1 ) tienen el mismo signo. Y esto es as ya que, por una parte, hemos visto
que x e y tienen signos opuestos cuando y = 0; y, por otra parte, como esta siempre
k+1
tambien son
en medio de dos convergentes consecutivos, los signos de pqkk y pqk+1
opuestos, es decir, pk qk y pk+1 qk+1 tienen signos opuestos. Por tanto, x(pk qk )
e y(pk+1 qk+1 ) tienen el mismo signo, como queramos.
Como primer corolario, podemos ver que el convergente k-esimo es la mejor aproximacion de un n
umero irracional mediante un n
umero racional de denominador como mucho
qk :
Corolario 7.12. Sea un n
umero irracional y sea pqkk el convergente k-esimo de su fraccion
continua. Entonces, si q qk , se tiene | pqkk | | pq | para cualquier entero p.
Demostraci
on: Como qk < qk+1 , se tiene, por el Lema 7.11, |pk qk | |p q| y por
tanto
pk
p
|pk qk |
|p q|
|p q|
| | =
= | |.

qk
qk
qk
q
q

Teorema 7.13. Sea un n


umero irracional y sea

p
q

<

1
2q 2 ,

entonces

p
q

es un convergente de .
76

p
q

un n
umero racional tal que q > 0. Si

Demostraci
on: Como qk es una sucesion creciente de n
umeros enteros, existira k tal que
p
pk
qk q < qk+1 . Veamos entonces que q = qk . Si no fuera as, entonces pqk qpk = 0,
luego |pqk qpk | 1. Tendremos entonces, por una parte:
1
|pqk qpk |
p pk

q
qk
qqk
qqk
y por otra parte, usando el Lema 7.11:
pk
|pk qk |
|p q|
p
p
p
p pk

+
+
+

=
q
qk
q
qk
q
qk
q
qk
=

q 1
1
1
q p
1
p
= 2+
.
+
< 2 +
2
q
qk q
2q
qk 2q
2q
2qqk

Poniendo juntas las dos desigualdades se obtiene


1
1
1
< 2+
,
qqk
2q
2qqk
lo que implica
eleccion de k.

1
2qqk

<

1
2q 2 ,

que a su vez implica q < qk , lo que es una contradicci


on con la

Corolario 7.14. Si p, q es una soluci


on positiva de x2 dy 2 = 1, entonces

convergente de d.
Demostraci
on: Por la Observaci
on 7.10, | pq

7.13, que pq es un convergente de d.

d| <

1
2q 2 ,

p
q

es un

lo que implica, por el Teorema

Necesitamos ver ahora que convergentes de = d son soluciones de la ecuacion de


Pell. Por la construcci
on por recurrencia del Ejemplo 7.2, est
a claro que cada k se puede

a una f
ormula precisa, en que lo
escribir en funci
on de d. El siguiente resultado nos dar

fundamental es que el coeciente de d en el numerador es 1:

Lema 7.15. Sea = d y, para cada k 0, sea k denido como en el Ejemplo 7.2.
Entonces, deniendo por recurrencia s0 = 0, t0 = 1,
sk+1 = ak tk sk
tk+1 =

d s2k+1
d (ak tk sk )2
=
tk
tk
77

se puede escribir k =

sk + d
,
tk

y ademas sk y tk son enteros y tk = 0.

Demostraci
on: Lo demostramos
por inducci
on sobre k, siendo claro el caso k = 0. Si

sk + d
ahora suponemos k = tk con sk y tk enteros y tk = 0, entonces tenemos
k+1 =

1
=
k ak

sk + d
tk

ak

tk ( d sk + ak tk )
tk

=
=
=
sk + d ak tk
( d + sk ak tk )( d sk + ak tk )

ak tk sk + d
sk+1 + d
tk (ak tk sk + d)
= d(a t s )2 =
.
=
k k
k
d (ak tk sk )2
tk+1
tk

Es claro que sk+1 es entero por serlo ak , sk , tk . Por otra parte, si escribimos
tk+1 =

d s2k+1
d (ak tk sk )2
d s2k
=
=
a2k tk + 2ak sk = tk1 a2k tk + 2ak sk
tk
tk
tk

se obtiene que tk+1 es entero (en realidad, como estamos usando que tk1 es entero, habra
que demostrar tambien el caso k = 1 de la inducci
on, pero al ser t0 = 1 es evidente que t1
es entero).
Proposici
on 7.16. Con las notaciones del Lema 7.15, se tiene:

pk
(i) Si qk es el convergente k-esimo de d, entonces p2k dqk2 = (1)k+1 tk+1 .
(ii) tk > 0 para todo k 0.
(iii) Existen enteros t, s tales que tk = t y sk = s para innitos valores de k.

Demostraci
on: Escribiendo d = [a0 ; a1 , . . . , ak , k+1 ], sabemos que coincide con su convergente (k + 1)-esimo, y por el Teorema 7.3 tendremos

k+1 pk + pk1
d=
=
k+1 qk + qk1

sk+1 + d
pk
tk+1

sk+1 + d
qk
tk+1

+ pk1
+ qk1

sk+1 pk + tk+1 pk1 + pk d

=
sk+1 qk + tk+1 qk1 + qk d

Por tanto


(sk+1 pk +tk+1 pk1 )+pk d = (sk+1 qk +tk+1 qk1 +qk d) d = dqk +(sk+1 qk +tk+1 qk1 ) d
lo que implica
pk =sk+1 qk + tk+1 qk1
dqk =sk+1 pk + tk+1 pk1
78

Usando estas igualdades junto con el Teorema 7.3(ii) tendremos


p2k dqk2 = pk (sk+1 qk + tk+1 qk1 ) qk (sk+1 pk + tk+1 pk1 ) =
= tk+1 (pk qk1 pk1 qk ) = (1)k1 tk+1
lo que prueba (i).
Demostremos (ii) por inducci
on sobre k. Es evidente para k = 0, ya que, por denici
on,
t0 = 1. Supongamos entonces que sabemos tk > 0 y demostremos tk+1 > 0. Por (i),
tendremos
2
p2k+1 dqk+1
tk+1
=
.
tk
p2k dqk2
El signo del numerador es igual al signo de
2
p2k+1 dqk+1
2
= Ck+1
d
2
qk+1

que es el signo de Ck+1 d. An


alogamente, el signo del denominador es el signo Ck d.

Como por el Teorema 7.6 los signos de Ck+1 d y Ck d son distintos, se sigue que
tk+1
en tk+1 es positivo. Esto
tk tiene signo positivo, y como tk es positivo entonces tambi
demuestra (ii).
Para demostrar (iii), basta recordar la igualdad tk tk+1 + s2k = d y, al ser tk , tk+1 > 0

por (ii), se tendr


a 1 tk d y |sk | < d. Por tanto, hay una cantidad nita de posibles
valores para los pares (tk , sk ), por lo que necesariamente alguno se debe repetir innitas
veces.

Observaci
on 7.17. Notese que la parte (i) de la Proposici
on 7.16 indica que la ecuaci
on
2
2
de Pell x dy = 1 tiene solucion (ya sabemos por el Teorema 7.13 que necesariamente
debe ser un convergente pqkk ) si y solo si existe alg
un k Z para el que tk+1 = 1 (recuerdese

que tk+1 > 0 por la parte (ii)). Esto es equivalente a decir k+1 = sk+1 + d, es decir,

1
1
= d[
= 1 . Por tanto, los
ak+1 = [k+1 ] = sk+1 + [ d] y k+2 = k+1 a
k+1
d]
datos de la fracci
on continua se deben repetir despues de k + 1 pasos, por lo que debe ser

d = [a0 ; a1 , a2 , . . . , ak+1 ].
De momento, la parte (iii) de la Proposici
on 7.16 garantiza s
olo que existen k, l tales
que tk+l = tk y sk+l = sk , y por tanto k+l = k . De aqu se deduce que ak +l = ak para
todo k k, y lo que faltara ver es que se puede tomar k = 1.

Lema 7.18. Para cada k 1, denimos k mediante la recurrencia 1 = a0 + d y


k+1 = ak + 1k . Entonces:
79

(i) k > 1 (y por tanto [k+1 ] = ak ).


(ii) k =

tk
dsk

sk + d
tk1 .

Demostraci
on: Para demostrar (i), usamos inducci
on sobre k, siendo evidente el caso
1
ormula de
k = 1. Si suponemos ahora k > 1, entonces k > 0, y como ak 1, de la f
1
recurrencia k+1 = ak + k se sigue k+1 > 1.
Para demostrar (ii), observamos primero que, por el Lema 7.15, tenemos:

tk (sk + d)
tk ( d + sk )
tk (sk + d)
sk + d
tk

=
=
=
=
d s2k
tk tk1
tk1
d sk
( d sk )( d + sk )
por lo que basta ver que k coincide con uno cualquiera de los dos valores. Lo demostramos
por inducci
on sobre. Si k = 1, recordando del Lema 7.15 las igualdades
s0 = 0, t0 = 1 y

s1 + d
s1 = a0 t0 s0 = a0 , tenemos inmediatamente 1 = a0 + d = t0 . Supongamos ahora
tk
demostrado k = ds
y demostremos la formula para k + 1. Para ello, aplicamos de
k
nuevo el Lema 7.15 y tendremos:

1
(ak tk sk ) + d
sk+1 + d
d sk
k+1 = ak +
= ak +
=
=
.
k
tk
tk
tk

Teorema 7.19. Si un entero positivo d no es un cuadrado perfecto, entonces su fracci


on

continua tiene la forma d = [a0 ; a1 , a2 , a3 . . . a3 , a2 , a1 , 2a0 ].


Demostraci
on: Como ya vimos en la Observacion 7.17, existir
an k, l tales que tk+l = tk y
sk+l = sk . Veamos en primer lugar que podemos tomar k = 1. En efecto, si k > 1, por la
parte (ii) del Lema 7.18 tendremos k+l = k , mientras que la parte (i) implica entonces
ak+l1 = [k+l ] = [k ] = ak1 . Como tambien tenemos
1
1
= k+l = k =
k+l1 ak+l1
k1 ak1
se concluye k+l1 = k1 , es decir, tk+l1 = tk1 y sk+l1 = sk1 . Reiterando el
proceso, llegaremos a tl+1 = t1 y sl+1 = s1 . Esto quiere decir que l+1 = 1 , lo que
implica que tendremos

d = [a0 ; a1 , a2 , . . . al ].
Por otra parte, podemos escribir:
1 = l+1 = al +

1
1
= al +
l
al1 +

= . . . = al +

1
l1

80

1
al1 +

1
al2 +

..

a2 +

1
a1 + 1
1

de donde se deduce 1 = [al ; al1 , al2 . . . , a1 ], o equivalentemente


1 = [al ; al1 , al2 . . . , a1 , al ].
Por otra parte tenemos tambien

1 = a0 + d = a0 + [a0 ; a1 , a2 , . . . al ] = [2a0 ; a1 , a2 , . . . al ].
Comparando ambas expresiones se obtiene el resultado.
En realidad, no nos va a interesar el aspecto simetrico que tiene el periodo, sino
simplemente saber que empieza en a1 y que longitud tiene.

Denici
on. Llamaremos longitud del periodo de d al mnimo entero positivo r para el

que podemos escribir d = [a0 ; a1 , . . . , ar ].

Teorema 7.20. Si el periodo de d tiene longitud r, entonces tk = 1 si y solo si r|k.

Demostraci
on: Como d = [a0 ; a1 , . . . , ar ], se sigue ir+1 = 1 para todo i, es decir,
1
1
=
.
ir air
d a0
De aqu sigue, junto con el Lema 7.15,

d a0 = ir air

sir + d
=
air
tir

luego quitando denominadores queda

(tir 1) d = sir air tir + a0 tir ,


de donde deducimos tir = 1 para todo i, es decir, que tk = 1 si r|k.
a k = sk +
Supongamos ahora tk = 1. Por tanto, por el Lema 7.15 se tendr
tomando parte entera se sigue

ak = [k ] = sk + [ d] = sk + a0 .
Por tanto,
k+1 =

1
1
1

=
=
= 1
k ak
(sk + d) (sk + a0 )
d a0

y por tanto el bloque a1 , . . . , ak se repite periodicamente en la fraccion continua de


decir, r|k.
El resultado nal es entonces el siguiente:
81

d, y

d, es


Teorema 7.21. Si el periodo de d tiene longitud r, entonces las soluciones positivas de
la ecuacion de Pell x2 dy 2 = 1 son:
(i) Si r es par, x = pir1 , y = qir1 .
(ii) Si r es impar, x = p2ir1 , y = q2ir1 .
Demostraci
on: Por el Corolario 7.14, las posibles soluciones de la ecuaci
on de Pell son

pk
on 7.16,
de la forma x = pk , y = qk donde qk es un convergente de d. Por la Proposici
2
2
k+1
k+1
x dy = (1)
tk+1 , por lo que debe ser (1)
tk+1 = 1. Como ademas tk+1 es
positivo, x = pk , y = qk es una solucion de la ecuacion si y solo si k impar y tk+1 = 1. Por
el Teorema 7.20, esto es equivalente a que k sea impar y r|k + 1. Distinguimos ahora dos
casos:
Si r es par, entonces la condici
on r|k + 1 ya implica que k es impar, luego x = pk ,
on si y s
olo si se puede escribir k = ir 1.
y = qk es una solucion de la ecuaci
Si r es impar, la condici
on r|k + 1 equivale a que se pueda escribir k = i r 1, y que
k sea impar equivale ahora a que i es par, luego se puede escribir k = 2ir 1.

82

Вам также может понравиться